Sie sind auf Seite 1von 78

NCLEX QUESTIONS

1) The nurse makes the following entry on an elderly clients care plan, Goal not met. Client refuses to ambulate, stating, I am too afraid I will fall. The nurse should take which of the following actions? (1) Notify the physician (2) Reassign the client to another nurse. (3) Reexamine the nursing orders. (4) Write a new nursing diagnosis. 2) Which of the following nursing diagnoses would most likely be associated with the absence of hair on a 70-year-old male clients legs? (1) Altered nutrition: less than body requirements. (2) Risk for infection (3) Fluid volume deficit (4) Tissue perfusion, altered: peripheral 3) A nurse is preparing to complete an admission assessment on a client that is partially hearing impaired. The best approach would be to do which of the following? (1) Request that a family member be present. (2) Prepare written questions that cover the assessment criteria. (3) Speak slowly in a low-pitched voice while facing the client. (4) Perform the physical assessment only at this time. 4) An elderly, terminally ill client is being cared for by her only daughter. The daughter expresses a fear of not knowing

how to care for her mother appropriately. Which nursing diagnosis is most appropriate? (1) Social isolation (2) Powerlessness (3) Situational low self-esteem (4) Altered role performance 5) A 76-year-old client is admitted to the hospital. The nursing assessment yields the following problems. Which of these problems is related to aging? (1) Hyperopia (2) Conductive hearing loss (3) Presbycusis (4) Tinnitus 6) Pain in the elderly requires careful assessment because older people have which of the following characteristics? (1) Increased pain tolerance (2) Decreased pain tolerance (3) Are likely to experience chronic pain (4) Experience reduced sensory perception 7) A nurse is working in a geriatric screening clinic. The nurse would expect that the skin of the normal elderly client will demonstrate which of the following characteristics? (1) Dehydration causing the skin to swell. (2) Moist skin turgor (3) Skin turgor showing a loss of elasticity. (4) Overhydration causing the skin to wrinkle

8) A 78-year-old visually impaired client is admitted to the nursing unit. Which of the following interventions would be most appropriate in reducing sensory deprivation? (1) Close curtains to reduce glare. (2) Keep doors open to provide bright light into the room (3) Keep curtains open to allow the sun to shine brightly in the room (4) Keep lights in the room dimmed. 9) Which of the following are associated with the aging process. Choose all that apply. (1) Increased visual impairment (2) Increased calorie requirements (3) Decreased sensation to heat and cold (4) Decreased subcutaneous fat

Which of the following factors are most likely to exacerbate risk for disease and frailty? (Select all that apply.)
Loss of appetite and delirium Sleep disturbances and failures of self-care Poor lifestyle choices and poverty Iatrogenesis and dizziness Risks for disease and frailty increase with age, and can be exacerbated by poor lifestyle choices and poverty.

Which of the following groups of older persons is more likely to have limitations in ADLs when chronically ill?

Which of the following is an accurate description of a frail older person?


A frail older person is one who is independent in minimum activities of daily living. A frail older person is one who has a noticeable planned weight loss of 10 pounds in a year. A frail older person is one who exhibits dependence in several activities of daily living. A frail older person is one who experiences frequent periods of depression. A frail older person is one who exhibits dependence in one or more activities of daily living. This dependency often restricts ADLs, and limitations in ADLs can contribute to feelings of depression.

Caucasians are more likely than AfricanAmericans to have limitations in ADLs when chronically ill. Native Americans are more likely than Caucasians to have limitations in ADLs when chronically ill. African-Americans are more likely than Caucasians to have limitations in ADLs when chronically ill. Hispanics are more likely than AfricanAmericans and Caucasians to have limitations in ADLs when chronically ill.

African-Americans are more likely than Caucasians to have limitations in ADLs when chronically ill.

Gerontology nurses have the challenge to educate the older population and their families that a healthy old age can be

achieved by:

Taking advantage of recommended preventive health services and lifestyle changes. Reducing their chances of disability with vigorous exercises and balanced diet. Becoming aware that cultural motivation is the key to achieving a healthy old age. A shift in paradigm as it relates to dietary intake, rest, and yearly medical interventions.

The gerontology nurse who is involved in health promotion programs for older persons living in the community and diagnosed with Alzheimer's disease will not focus on which of the following?
Assisting in the ability to make appropriate, informed choices Achieving optimal mental and physiological health Providing information and appropriate skills to maintain lifestyle changes Reducing genetic risk factors related to comorbidities of the older person When cognitive capacity is impaired due to the diagnosis of Alzheimer's disease or another irreversible neurogenerative dementia, care and treatment decisions become even more difficult. Treatment choices are not simple, because the burden of treatment and the success of needed interventions requires a skilled interdisciplinary team approach.

Americans can improve their chances for a healthy old age by simply taking advantage of recommended preventive health services and by making healthy lifestyle changes.

The established goal of the National Institute on Aging is to:

Provide appropriate healthcare to the chronically ill, so as to lengthen their years. Decrease the significant burden of care placed on gerontology nurses and other healthcare providers. To "add life to years" by educating professionals and supporting appropriate research. To help promote sophisticated levels of interdisciplinary care and "fine-tune" appropriate therapies.

Which of the following classifications of drugs would not directly cause delirium through intoxication or withdrawal?
Anesthetics Analgesics Corticosteroids Cardiotonics Medications that can cause delirium through intoxication or withdrawal include anesthetics, analgesics, anti-asthmatics, anticonvulsants, antihypertensives, antimicrobials, corticosteroids, gastrointestinal H2 blockers, muscle relaxants, hypnotics, and psychotropic medications.

The established goal of the National Institute on Aging is to "add life to years" and to educate professionals and support research that establishes specific, practical ways to reduce disability. The National Institute on Aging does not provide direct healthcare service to individuals. The agency's mission is to support and conduct high-quality research, to train and develop highly skilled research scientists, to develop and maintain state-of-the-art resources, and to disseminate information to the public.

Which of the following statements accurately describes modified aggressive treatment for persons confined to nursing homes?
Modified aggressive treatment is usually appropriately delivered to older persons with high functional ability. Modified aggressive treatment is usually appropriate for older persons with higher degrees of frailty or multiple co-morbidities who still have sufficient reserve capacity. Modified aggressive treatment is appropriate for all older persons, and can be delivered alone or in conjunction with aggressive or modified care. Modified aggressive treatment is delivered to those with a life expectancy of six months or less; patient's age is not a factor. Modified aggressive treatment is usually appropriate for older persons with higher degrees of frailty or multiple co-morbidities who still have sufficient reserve capacity to respond to the demands of treatment.

Healthcare providers can practice preventive ethics in order to promote an environment where early identification of issues and anticipation of possible dilemmas can serve to proactively avert potential areas of conflict. The knowledge of the values of the patient and family will allow the healthcare provider to anticipate where patient and family values might be in conflict.

Which of the following statements is accurate pertaining to the immune dysregulation theory?

he three characteristics that are unique to the immune system are self-recognition, specificity, and memory. Antigens are a wide range of substances that are identified as "non-self," and stimulate an immune response. The three characteristics that are unique to the immune system are self-recognition, specificity, and memory. Antigens are a wide range of substances that are identified as "non-self," and stimulate an immune response.

Because persons with end-stage dementia and physical frailty are in need of a Early identification of issues to help avert rational approachof conflict.at the end of life, potential areas to care preventive knowledge of the values of patient and Having ethics to promote a therapeutic environmentrelationship to healthcare. family in include:
Avoiding conflict resolution and communication skills with extended families. Ongoing aggressive treatment of all disease symptomatology to avert shortening of the elderly lifespan.

The defense of natural immunity, or innate immunity, is not produced by the immune response that is present at birth. Defenses of the innate immune system consist of a variety of physical barriers and biochemical and cellular defense mechanisms to prevent the establishment of potential pathogens within the body.

Which of the following factors affect proper immune system function?


Age, exercise Sunlight, nutrients in the diet Diagnosis with co-morbidities All of the above Aging results in lifetime accumulative effects of environmental exposures, such as sunlight, radiation, pesticides, and the amount of illness and stress the aged persons have been exposed to.

The cellular immune response is directly related to the T cell surface receptors. The humoral immune response is indirectly related to the T cell surface receptors. The cellular immune response is indirectly related to the B cell surface receptors. The humoral immune response is initiated when an antigen binds with antibody receptors on the surface of the mature B cell. In the cell-mediated immune response, T cells bind to the foreign antigen in multiple areas on the surface of the cell.

The gerontology clinical nurse specialist is teaching a group of new graduate nurses how the body fights infection. When discussing humoral immunity, the nurse should explain that this protection is mediated by which cell?
Basophil Eosinophil The B cell The monocyte The humoral immune response is initiated when an antigen binds with antibody receptors on the surface of the mature B cell.

A nurse is caring for a patient who is deficient in T cells. The nurse would know that T cell deficiency would result in which of the following?
Hypersensitivity to allergens Impaired cellular immunity A decrease in the cell lifespan of granulocytes A cellular inability to participate in phagocytosis T cells act directly with cellular targets, and are responsible for cell-mediated immunity.

Which nursing intervention should be attended to first for a client with a Type I hypersensitivity reaction?
Administering naloxone (Narcan) stat Obtaining blood sample for type and crossmatch Initiating 0.9% NaCl intravenous solution Establishing and maintaining a patent airway

Which of the following statements are accurate about the humoral immune response and the cellular immune response? (Select all that apply.)

The humoral immune response is directly related to B cell surface receptors.

Type I hypersensitivities are immediate, and can be lifethreatening. Manifestations include tightening of the throat, shortness of breath, wheezing, tachycardia, and hypotension.

hypersensitivity that affects older people. Antibodies form against the person's own IgGs, and the resulting complex is identified as foreign.

Which of the following clinical manifestations of a patient suggests Type IV hypersensitivity reaction?
Dermatitis from a latex allergy Sudden shortness of breath The appearance of hives Swelling of the lips Type IV hypersensitivity is also called delayed hypersensitivity. Tissue is damaged as a result of a delayed T cell reaction to an antigen. Examples of Type IV hypersensitivity include dermatitis from a latex allergy, tuberculin reactions, and transplant rejections.

Which of the following classifications of drugs are usually prescribed for patients with rheumatoid arthritis?
Non-steroidal anti-inflammatory (NSAIDs) Glucocorticoids Mineralocorticoid Answers 1 and 2 Drug therapy often consists of a combination of non-steroidal anti-inflammatory drugs (NSAIDs), anti-rheumatic drugs, and glucocorticoids.

Under which of the following classifications of hypersensitivity would systemic lupus erythematosus (SLE) and rheumatoid arthritis be classified?
Type I hypersensitivity Type II hypersensitivity Type III hypersensitivity Type IV hypersensitivity Systemic lupus erythematosus is an example of type III hypersensitivity reaction caused by auto-antibody production. The antibodies are formed against nuclear DNA and RNA throughout the body. Rheumatoid arthritis is another example of type III

Which of the following are unique characteristics associated with human immunodeficiency virus (HIV) infection in the older person?
HIV infection in the older person most likely is underdiagnosed and underreported. Healthcare workers often do not take sexual histories of older patients or recommend HIV testing. Symptoms of memory loss and weight loss related to HIV often are treated as common age-related health problems. Older persons are contacting HIV infection through heterosexual and homosexual activities.

The older person's susceptibility to infections is related to: (Select all that apply.)

A decline in responsiveness of the immune system to harmful foreign invaders. The febrile response that signals infections sometimes being blunted in the older person. Medications commonly taken that also decrease the normal fever response. The body temperature in older people being approximately 1F lower than normal, meaning a rise in temperature can go unnoticed.

(B) delirium (C) functional psychosis (D) mania (E) organic delusional syndrome 2. The percentage of community-dwelling individuals older than 65 years who require assistance with bathing, dressing, walking within their homes, using the toilet, or eating is closest to (A) 10% (B) 30% (C) 50% (D) 70%

UCLA Geriatrics Knowledge Test DIRECTIONS: Each of the questions or incomplete statements below is followed by suggested answers or completions. Select the ONE that is BEST in each case. 1. One day after operative repair of a left intertrochanteric fracture of the femur, an 83-yearold man is agitated, confused, and appears to be having visual hallucinations. He has accused the nursing staff of trying to kill him. He is a widower and has lived alone for 11 years. He has been forgetful for the past year but can perform all activities of daily living without assistance. On mental status examination he is easily distracted, perseverates, and does not respond coherently to questions about orientation and memory. The most likely diagnosis is (A) Alzheimers disease

3. A 78-year-old nursing home resident has mild dementia associated with Alzheimers disease. She is disoriented to time and place but knows family members and regular nurse aides by name. This patients capacity to make decisions regarding her health care is best determined by (A) mental status test (B) her ability to understand treatment options (C) probate court decision (D) psychiatric examination 4. Which of the following instruments is most appropriate to assess physical functional

abilities in a 75-year-old nursing home resident? (A) Activities of Daily Living Scale (B) Face/Hand Test (C) Instrumental Activities of Daily Living Scale (D) Zung Self-Rating Depression Scale 5. An 85-year-old man has a 4x7-cm Shea stage 3 pressure ulcer over the sacrum. He has been confined to bed since sustaining a hip fracture three months ago and has lost 10 lb during this time. Examination shows a foul-smelling wound, necrotic tissue covering 50% of the ulcer, and purulent drainage at the base. There is no cellulitis. In addition to surgical debridement, the most appropriate initial management is (A) topical application of neomycin/polymyxin/bacitracin ointment (B) topical application of sodium hypochlorite solution (C) application of an adhesive polyurethane dressing (D) myocutaneous flap (E) none of the above 6. An 82-year-old nursing home resident has had a 7-lb weight loss over the past two months. She had a stroke seven years ago that resulted in aphasia. She is bed bound and needs assistance when eating. The staff reports that she often clamps her teeth and refuses to eat. Physical examination shows no abnormalities. The most appropriate next step in

management is (A) observation (B) contact the family (C) begin nasogastric tube feeding (D) admit to the hospital (E) perform percutaneous endoscopic gastrostomy 7. A 78-year-old woman has right hemiplegia and dysarthria after a cerebral infarction in the left pons three weeks ago. She has been fed through a gastrostomy tube for two weeks because of pneumonia of the left lower lobe, presumed to be secondary to aspiration. She is depressed and would like to begin oral feedings. Examination shows a decreased gag reflex and mild dysarthria. The most appropriate management is (A) continued tube feedings until gag reflex returns to normal (B) encouragement to eat an appetizing diet (C) ice chips orally and antidepressant therapy (D) supervised oral feeding of pureed foods (E) swallowing videofluoroscopy 8. The memory deficit that is most commonly caused by depression is (A) amnesia for traumatic events (B) difficulty with free recall (C) difficulty with multiple choice (D) disorientation to person, place, and time (E) symmetrical loss of multiple cognitive functions

9. Which of the following is best used for routine evaluation of nutritional status in older persons? (A) Delayed skin testing (B) Serial weight measurements (C) Serum albumin level measurement (D) Serum transferrin level measurement (E) Total lymphocyte count 10. The most common community-acquired infection that results in hospitalization in older persons is (A) infected pressure ulcers (B) influenza (C) pneumonia (D) tuberculosis (E) urosepsis 11. Approximately what percentage of the cost of long-term care is paid for directly by elderly persons or their families? (A) 30% (B) 50% (C) 70% (D) 90% 12. An 82-year-old man has worsening of urge incontinence, urinary frequency, and nocturia three months after transurethral resection of the prostate for benign prostatic hyperplasia.

Symptoms of obstruction, including hesitancy, straining, and intermittent urinary stream, have improved. He is otherwise healthy and takes no medications. Urinalysis shows normal findings. After voiding 220 mL without straining, postvoid residual volume is 40 mL. The most appropriate next step in management is (A) oxybutynin 2.5 mg three times daily for a two-week trial (B) terazosin 0.5 mg twice daily for a two-week trial (C) cystoscopy (D) cystometrography 13. An 82-year-old woman with Alzheimers disease sustains a displaced fracture of the femoral neck in a fall. She resides in a nursing home because of her cognitive deficits. Prior to the injury, she was independent in ambulation, feeding, toileting, and transferring, but needed cueing to bathe and dress. Her score on the MiniMental State Examination is 11/30. The most appropriate management of the fracture is (A) use of crutches and limited mobility (B) traction and bed rest (C) open reduction and internal fixation (D) prosthetic replacement of the femoral head 14. A 70-year-old man with diabetes mellitus and emphysema comes to your office in December for influenza vaccination. You have cared for this patient for the past six

months and no additional medical records are available. He reports that prior to your care, his last visit to a physician was 10 years ago. He also reports that he received a tetanus shot in the emergency department four years ago because of a laceration. Which of the following is the most appropriate schedule for immunization? (A) Influenza vaccine only at this visit (B) Influenza and pneumococcal vaccines at this visit (C) Influenza vaccine at this visit and pneumococcal vaccine in eight weeks (D) Pneumococcal vaccine at this visit and influenza vaccine in eight weeks 15. A 76-year-old woman has pain in the right hip and is unable to walk without support three days after a fall. Passive motion of the right hip produces pain in the groin. Plain radiographs of the pelvis show no abnormalities. The most likely cause of her symptoms is (A) muscle strain (B) occult fracture of the hip (C) osteoarthrosis (D) referred pain from the spine 16. Which of the following is required for eligibility for the Medicare Hospice Benefit? (A) Diagnosis of a terminal malignancy (B) Enrollment in Medicare Part A (C) Forfeiture of all Veterans benefits (D) Supplemental insurance policy

(E) Verification of need by two independent physicians 17. What percentage of persons who are age 65 years and older will require nursing home care at some time during their life? (A) 5% (B) 25% (C) 45% (D) 65% (E) 85% 18. A 79-year-old woman has had confusion since admission to the hospital for pneumonia one week ago. Her prior cognitive status is unknown; she appeared malnourished and unkempt on admission. She has responded well to antibiotic therapy and is currently afebrile. She is pleasant and attentive to questions. Mental status examination reveals disorientation, memory impairment, and language abnormalities; she is unable to draw the face of a clock. The remainder of the physical and neurologic examination shows no abnormalities. The most likely diagnosis is (A) age-associated memory loss (B) delirium (C) dementia (D) depression (E) paraphrenia

19. A 68-year-old man has a moderate left hemiparesis and severe neglect six days after a right hemispheric stroke. He appears to be in denial about his degree of impairment and states that he hopes to be discharged from the acute hospital to his home where he has lived alone for the past 10 years. During physical therapy sessions, he is apathetic and tires easily after 30 minutes. Further evaluation shows no abnormalities. Which of the following is the most appropriate site for rehabilitation? (A) Home-care program (B) Outpatient physical therapy program (C) Acute inpatient rehabilitation program (D) Nursing home (E) Inpatient geropsychiatric unit 20. An 84-year-old man is fitted for and trained in the use of a hearing aid. Three months later, he reports that he has stopped using the hearing aid because, although he can hear better, he still cannot understand what people are saying. He states that background noise is also a problem. He has hypertension but is otherwise healthy. His score on the Mini-Mental State Examination is 30/30. The most likely cause of the hearing difficulty is (A) central auditory processing disorder (B) early signs of dementia (C) improper fit of the hearing aid (D) Menieres disease

(E) noncompliance 21. An otherwise healthy 73-year-old man with Parkinsons disease has difficulty walking. He has had many changes in his antiparkinsonian medication and feels that the present regimen is the best he has ever had. Analysis of gait reveals very slow step initiation, decreased stride length with shuffling, and moderate retropulsion. Which of the following devices is most likely to improve his gait? (A) A straight cane (B) A four-prong quad cane (C) A walker with arm troughs attached (D) A pick-up four-legged walker (E) A front-wheeled walker Items 22-23 For each patient, select the most appropriate management (AL). (A) Pelvic muscle exercises (B) Prompted voiding (C) Biofeedback (D) Urodynamic assessment (E) Oxybutynin 2.5 mg three times daily (F) Phenylpropanolamine 75 mg twice daily (G) Vaginal estrogen daily (H) Terazosin 0.5 mg twice daily (I) Bethanechol 10 mg three times daily (J) Bladder neck suspension (K) Intermittent catheterization (L) Chronic indwelling catheterization

22. An 82-year-old woman is admitted to a nursing home two weeks after operative repair of a hip fracture. She has an indwelling catheter for recurring postoperative urinary retention. 23. An 89-year-old woman with Alzheimers disease, mild congestive heart failure, and glaucoma has urinary incontinence several times daily, occurring both day and night and without residual urine. She is wheelchair-bound and resides in a nursing home.

3. The progression of intermittent claudication is measured by the: 1. distance walked before leg pain starts. 2. pedal edema after dangling the legs for 20 minutes. 3. peripheral pulses in the affected leg. 4. skin temperature and color of the feet. Correct Response: 1 4. An early sign of alcohol withdrawal is: 1. auditory hallucinations. 2. decreased blood pressure. 3. depression. 4. diaphoresis. Correct Response: 4 5. A stage III pressure ulcer is characterized by: 1. blisters, abrasions, or shallow craters. 2. deep craters with or without undermining and full-thickness skin loss involving subcutaneous tissue. 3. full-thickness skin loss with tissue necrosis or damage to muscle or bone. 4. partial-thickness skin loss involving the dermis or epidermis. Correct Response: 2 6. The primary risk factor for the development of pressure ulcers in older adult patients is: 1. immobility. 2. impaired circulation. 3. incontinence. 4. malnutrition.

1. For an individual with age-related hearing loss, which sound is most difficult to hear? 1. A recording of a march played softly 2. A young child talking in a cafeteria line 3. Hammering during construction of a house next door 4. The voice of a man speaking in an elevator Correct Response: 2 2. Pain of gastrointestinal origin is best differentiated from pain of cardiac origin by the presence of: 1. chest pain lasting longer than five minutes. 2. chest pain of rapid onset. 3. left flank pain. 4. substernal chest discomfort. Correct Response: 4

Correct Response: 1 7. An 80-year-old resident of a retirement center states that something is wrong with the lighting in the room because colored rings appear around the light bulbs. The resident most likely has: 1. cataracts. 2. delusions. 3. glaucoma. 4. increased intracranial pressure. Correct Response: 3 8. The most common cause of chronic pain in older adults is: 1. arthritis. 2. fractures. 3. headaches. 4. neuropathy. Correct Response: 1 9. A 72-year-old man with asthma, chronic obstructive pulmonary disease, and chronic anxiety is admitted to a nursing home. Care plan objectives for this man include: 1. adherence to his medication regimen, inhalation therapy, and instruction about methods of conserving energy. 2. an exercise program to increase the vital capacity of his lungs. 3. instruction in respiratory exercises with emphasis on forced inhalation.

4. oxygen therapy at 3 L/min as needed and deep-breathing exercises for relaxation. Correct Response: 1 10. A 65-year-old man with a 45-year history of smoking complains of a change in his cough pattern, a nonproductive cough, and an ache in his chest. The man's chest x-ray reveals an infiltrate. The gerontological nurse suspects: 1. a lung tumor. 2. chronic obstructive pulmonary disease. 3. pulmonary edema. 4. tuberculosis. Correct Response: 1 11. Which symptom in older adults is most indicative of a urinary tract infection? 1. Confusion 2. Dysuria 3. Fever 4. Frequency Correct Response: 1 12. Which is a risk factor for vaginitis in older adult women? 1. Anticoagulation therapy 2. Increased sexual activity 3. Poor nutrition 4. Prolonged antibiotic therapy Correct Response: 4

13. Which condition might be indicated by an increase in hemoglobin values? 1. Dehydration 2. Infection 3. Malnutrition 4. Opiate use Correct Response: 1 14. An 83-year-old female patient underwent a total hip replacement three days ago. She should not cross her legs because: 1. abduction of the hip can cause dislocation of the prosthesis. 2. adduction of the hip can cause dislocation of the prosthesis. 3. blood clots often result from pressure on arteries in the legs. 4. contracture can be prevented by avoiding acute flexion of the hip. Correct Response: 2 15. A 76-year-old patient with osteoarthritis complains of pain, stiffness, and deformities of the fingers. The gerontological nurse recommends: 1. cold packs. 2. exercise. 3. meditation therapy. 4. vitamin therapy. Correct Response: 2 16. A factor that contributes to hypothermia in older adults is: 1. decreased activity. 2. decreased vulnerability to cold. 3. increased perception of cold. 4. increased subcutaneous fat.

Correct Response: 1 17. Heat stroke is a serious form of hyperthermia that is characterized by: 1. absence of sweating. 2. decrease in body temperature. 3. increase in sweating. 4. nausea and vomiting. Correct Response: 1 18. A gerontological nurse is teaching dressing techniques to a patient with right hemiplegia and a severe speech impairment. An appropriate first step in the nurse's teaching would be to: 1. ask the patient to put on a shirt. 2. demonstrate the proper way to put on a shirt. 3. explain the difficulties in putting on a shirt. 4. give verbal instructions on dressing procedures. Correct Response: 2 19. Older adults with Parkinson disease exhibit: 1. confusion and depression. 2. dementia and hyperkinesia. 3. rigidity and tremor at rest. 4. weakness and tremor with movement. Correct Response: 3 20. To prevent injury, a gerontological nurse advises an older adult who is taking tricyclic antidepressants to:

1. eat a diet high in roughage. 2. get an additional night light. 3. provide lubrication for the oral mucosa. 4. stand up slowly from sitting or lying positions. Correct Response: 4 21. Persons who are taking anticholinergic drugs are at high risk for: 1. cardiac arrhythmias. 2. dry mouth. 3. orthostatic hypotension. 4. slurred speech. Correct Response: 2 22. Age-related changes in which two organs most affect an older adult's reaction to medication? 1. Heart and lungs 2. Intestines and spleen 3. Liver and kidneys 4. Pancreas and gall bladder Correct Response: 3 23. Sildenafil citrate (Viagra) is hazardous for patients with: 1. a history of coronary artery bypass graft. 2. a history of shingles. 3. heart failure and borderline hypotension. 4. Paget disease and hypertension. Correct Response: 3

24. A 78-year-old patient calls a telephone triage nurse and reports several falls after taking over-the-counter medication for a recent cold. Which medication contributed to the falls? 1. Diphenhydramine (Benadryl) 2. Ferrous sulfate 3. Guaifenesin (Robitussin) 4. Loratadine (Claritin) Correct Response: 1 25. Older adults who take the herbal supplement cascara sagrada are monitored for the presence of: 1. fever. 2. hypokalemia. 3. jaundice. 4. vertigo. Correct Response: 2 26. A nursing home conducts a survey to evaluate nursing care. However, some residents did not express their concerns due to fear of reprisal by the staff. Which aspect of the evaluation is most affected? 1. Generalizability 2. Reliability 3. Statistical significance 4. Validity Correct Response: 4

27. A common side effect of angiotensin-converting enzyme inhibitors that frequently results in discontinuation of therapy is: 1. a dry, persistent cough. 2. exacerbation of heart failure. 3. sedation. 4. urinary incontinence. Correct Response: 1 28. A patient with Stage 2 Alzheimer disease visits the mental health clinic. During the interview, the patient becomes hostile and refuses to answer further questions. The gerontological nurse's best action is to: 1. ask if the questions upset the patient in any way. 2. discontinue the interview. 3. explain that the information is needed to plan the patient's care. 4. ignore the patient's reaction and proceed. Correct Response: 2 29. A gerontological nurse at a nursing home conducts a reminiscence therapy group for residents with confusion. A member of the group stands up and says, I just heard my cow. I have to go and milk her now. The nurse's most therapeutic response is: 1. All right, you may leave the group now. 2. Please tell us about your cow. 3. That wasn't a cow; maybe you heard a vacuum cleaner. 4. You live here at the nursing home now, not on the farm. Correct Response: 2

30. Reminiscence therapy promotes an older adult's sense of security by: 1. increasing socialization skills. 2. meshing the past with the future. 3. providing praise and recognition. 4. reviewing comforting memories. Correct Response: 4 31. A physician has just informed an older adult patient that test results indicate that the patient has cancer and will require extensive surgery. The patient says, I know the tests are wrong. I feel fine. The gerontological nurse's most appropriate response is to: 1. acknowledge that the patient looks healthy and encourage seeking a second opinion. 2. advise the patient to join a support group. 3. convey availability to talk to the patient. 4. tell the patient that the tests are reliable and accurate. Correct Response: 3 32. A 75-year-old patient who sustained a stroke has residual left-sided weakness. From the first day of hospitalization, the patient has been combative and demanding, and has refused to swallow any medication. The most constructive nursing action is to: 1. continue to attempt to follow the physician's orders. 2. determine the patient's premorbid personality.

3. restrain the patient and request a change in the route of medication. 4. wait for the patient to become more cooperative. Correct Response: 2 33. A healthy 80-year-old female patient complains that her skin feels dry and sometimes itchy. The gerontological nurse advises her to: 1. avoid scratching since breaks in the skin increase the risk of infection. 2. drink more liquids and take showers instead of baths. 3. take fewer baths, use soap sparingly, and apply skin cream afterward. 4. wear cotton clothing and try a different brand of soap. Correct Response: 3 34. A 90-year-old patient with multiple medical problems is admitted to the hospital's geriatric care unit. The nursing assessment reveals lethargy, poor capillary perfusion, and urinary incontinence. These findings alert the gerontological nurse to the potential for: 1. aspiration. 2. contractures. 3. dehydration. 4. skin breakdown. Correct Response: 4 35. An 80-year-old patient complains of sleeping less despite spending more time in bed.

The patient does not use alcohol, caffeine, or any medications other than acetaminophen for arthritis pain. The patient goes to bed at 11:00 pm, falls asleep in 15 minutes, awakens several times during the night, and promptly goes back to sleep. The patient feels refreshed in the morning and works five days a week as a volunteer. The gerontological nurse's most appropriate response is to: 1. recommend that the patient eliminate fluid intake after 6:00 pm. 2. recommend that the patient go to bed one hour earlier. 3. suggest that the patient enroll in a sleep study. 4. tell the patient that the sleeping pattern is a normal agerelated change. Correct Response: 4 36. Which signs are characteristic of deep vein thrombosis? 1. A cool, non-tender limb 2. Limb numbness with diaphoresis 3. Rapid onset of unilateral leg swelling with dependent edema 4. Vertigo with an abrupt onset of blurred vision Correct Response: 3 37. Unusual symptoms of pneumonia often found in long-term care residents include: 1. anorexia and new behavioral problems. 2. headache and difficulty breathing. 3. muscle aches and fever. 4. nonproductive cough and chest pain. Correct Response: 1

38. Which symptom is exhibited first by an older adult with a urinary tract infection? 1. Anorexia 2. Confusion 3. Fever 4. Restlessness Correct Response: 2 39. Which assessment tool measures economic resources, mental health, and activities of daily living? 1. Lawton's Physical Self-Maintenance Scale 2. Older Americans Resources and Services Assessment 3. Problem Oriented Medical Record 4. Short Portable Mental Status Questionnaire Correct Response: 2 40. All care plans for older adult patients include: 1. a bowel and bladder program. 2. a fall prevention program. 3. discharge planning. 4. reminiscence therapy. Correct Response: 3 41. An older adult patient, who is recovering from surgery, has a sodium level of 128 mEq/L and is confused. The physician diagnoses syndrome of inappropriate secretion of antidiuretic hormone. The gerontological nurse's primary goal for this patient is to: 1. decrease edema by restricting free water intake. 2. prevent complications of hyponatremia. 3. reorient the patient to his or her surroundings.

4. restore the patient's fluid and electrolyte balance. Correct Response: 4 42. The adult children of an aging couple ask a gerontological nurse about alternative therapies or nontraditional services to improve their parents' health. The nurse's most appropriate response is to: 1. discuss the benefits, risks, and limitations of various therapies. 2. distinguish between folk and traditional medicine. 3. give a firm warning about alternative therapies. 4. recommend a reputable holistic health therapist. Correct Response: 1 43. A 73-year-old patient is admitted to a rehabilitation facility after sustaining a mild stroke. After three nights in the facility, the patient begins to sleep only four to five hours a night and to awaken frequently during the night. The patient then complains of not feeling rested and begins to nap during the day. Which is the most appropriate nursing action? 1. Completing an assessment of the patient's sleep-wake cycle to determine necessary interventions 2. Doing nothing since this type of sleep pattern is associated with normal aging 3. Inquiring if the patient takes a medication at bedtime and requesting that the physician order it

4. Moving the patient further away from the nurse's station to minimize disturbances Correct Response: 1 44. A 68-year-old man calls his daughter every night to talk about his beloved wife who died four weeks ago. During the day, he is sad and goes out frequently to get away from the empty house. The man's most probable state is: 1. depression. 2. mourning. 3. neurosis. 4. prolonged grief. Correct Response: 2 45. The main reason that older adults with a chronic illness tolerate functional impairments is that they: 1. are afraid to seek medical advice for fear of what they may find. 2. associate their symptoms with aging rather than an illness. 3. believe in home remedies and parental traditions. 4. relate their symptoms to conditions that resolved in the past without treatment. Correct Response: 2 46. An 87-year-old man, who has been living independently, is entering a nursing home. To help him adjust, the most effective action is to: 1. involve him in as many activities as possible so he can meet other residents.

2. move him as quickly as possible so that he does not have time to think. 3. restrict family visits for the first two weeks to give him time to adjust. 4. suggest that he bring his favorite things from home to make his room seem familiar. Correct Response: 4 47. An 80-year-old patient is in the terminal stage of Alzheimer disease. The treatment team meets with the patient's who are angry and complain about a recent incident in which their parent's dentures were misplaced. The team members realize that: 1. anger at staff is a symptom of grief and needs to be addressed. 2. anxiety about the meeting may have interfered with the children's affect. 3. material items are the focus for the children at this time. 4. the focus of control should shift to the parent. Correct Response: 1 48. When relocating from a family home to a continuing care community, the most crucial factor in an older person's adjustment is his or her: 1. level of economic independence. 2. perceived control of the move. 3. physical proximity to remaining family. 4. risk-taking ability. Correct Response: 2

49. A 78-year-old male resident at a long-term care facility, who is a former business executive, has been smoking and extinguishing cigarettes in a paper cup in areas where smoking is prohibited. He has been informed repeatedly of the designated smoking areas. The resident's behavior indicates an attempt to: 1. express self-transcendence. 2. maintain autonomy by exercising control. 3. maintain his previous professional role. 4. react against the facility's ageism. Correct Response: 2 50. A gerontological nurse is caring for an older adult who has been confined to home for the last 10 weeks due to illness. The patient is anxious, has multiple somatic complaints, and has become unable to follow instructions. The nurse knows that this phenomenon commonly occurs with: 1. environmental overload. 2. protective isolation. 3. selective inattention. 4. sensory deprivation. Correct Response: 4 51. The gerontological nurse facilitates the benefits of life review by: 1. assisting the older adult to accept death as the inevitable last stage.

2. changing the topic when a patient talks about his or her morbid past. 3. encouraging reminiscence, oral histories, and storytelling. 4. helping the older adult explore how spiritual involvement assists with stress relief. Correct Response: 3 52. The holiday season is approaching and a woman is admitted to a psychiatric unit. She reports that her husband of 45 years passed away four weeks ago. She frequently cries, eats poorly, periodically complains of back and stomach aches, and has begun isolating herself. What type of grief is this woman exhibiting? 1. Acute 2. Anticipatory 3. Disenfranchising 4. Dysfunctional Correct Response: 1 53. A 92-year old patient, who recently underwent a below-theknee leg amputation, is resisting attempts at rehabilitation. The most likely reason for the resistance is that the patient: 1. has goals that differ from the rehabilitation care plan. 2. has too many disabilities to realistically plan for rehabilitation. 3. is too old to undergo rehabilitation. 4. requires a psychiatrist's assistance to deal with the loss. Correct Response: 1

54. When the gerontological nurse in a clinic asks a 70-yearold man about his sexual activity, he begins to cry and says, I feel so bad for my wife; she is only 60, and I can't seem to satisfy her as often as I used to. Which is the nurse's most appropriate response? 1. At your age, sexual activity diminishes because of changes in your circulation. I will explain this to your wife. 2. Certain body functions, such as erections, slow down with age. Could you tell me more about your sexual relations? For instance, how often do you have intercourse? 3. Your problem is probably an emotional one. If you could relax, you would be as sexually active as you were 10 years ago. 4. Your problem is probably due to a decrease in your sexual hormones. This occurs naturally as people age. Correct Response: 2 55. An alert and oriented 82-year-old woman, who lives with her daughter, has been admitted to the hospital with bruises about the face and head. The daughter reports that her mother fell. Which behavior by the daughter raises the greatest suspicion of elder abuse?

1. Becoming defensive when questions are asked 2. Complaining about care delivered by hospital staff 3. Giving an illogical account of her mother's fall 4. Refusing to leave her mother alone to answer questions Correct Response: 4 56. In teaching an older adult client, the gerontological nurse's most appropriate initial strategy is to: 1. assess the client and individualize the teaching methods. 2. set a slow learning pace and begin teaching simple concepts. 3. teach slowly and use repetition. 4. use demonstration and provide ample opportunity for practice. Correct Response: 1 57. In preparing a presentation for older adults, a gerontological nurse keeps in mind that: 1. older adults are unlikely to participate in educational programs due to lack of interest. 2. older adult learners are heterogeneous due to diverse educational experiences and learning strategies. 3. the ability to acquire knowledge from a verbal presentation decreases with age more than the ability to acquire knowledge through reasoning. 4. the age-related decline in intellectual performance creates obstacles for acquiring new information. Correct Response: 2

58. Members of a family are caring for their father at home. Which statement by a family member indicates a need for teaching and caregiver instruction? 1. Dad has gotten lazy about his bathroom habits. He blames his arthritis medication for his toileting accidents. 2. Dad's room is close to the bathroom and we keep a light on for him at night. 3. It's inconvenient, but we stop other activities to remind Dad to go to the bathroom on a regular schedule. 4. We try to avoid coffee and tea at night, but Dad really likes a cup of coffee for breakfast. Correct Response: 1 59. When hospital quality assurance indicators are assigned, the nursing staff strives to: 1. avoid emphasizing performance deficits. 2. decrease patient falls. 3. focus on achieving 100% compliance. 4. identify high-risk, high-volume, or problem-prone areas. Correct Response: 4 60. The most appropriate environment for a person with chronic dementia is one that: 1. changes often to decrease boredom. 2. contains familiar objects. 3. is limited in color and sound.

4. is stimulating so as to challenge thought. Correct Response: 2 61. In assessing the lighting for a patient with glaucoma, the gerontological nurse knows that: 1. ceiling lights are best. 2. drapes should be left open during the daylight hours. 3. higher levels of light are needed. 4. lower levels of light are needed. Correct Response: 4 62. A 63-year-old patient is returning home after being hospitalized for injuries received during a robbery and home invasion. Although neighborhood robberies are rampant, the patient has lived in the same house for 50 years and does not want to move. The patient receives a monthly social security check. The gerontological nurse's most appropriate step is to: 1. advise the patient to arrange for someone to visit regularly or move into the home. 2. advise the patient to have the social security check deposited directly to the bank and to get a dog. 3. assume that the patient is incompetent and initiate commitment proceedings. 4. take no action since the patient has a right to autonomy. Correct Response: 2

63. A gerontological nurse in a daycare program for older adults observes that the participants have long toenails, corns, calluses, and other problems indicating a need for better foot care. What is the nurse's best action? 1. Developing an educational program on foot health and arranging for podiatry services at the site 2. Establishing a regular foot care plan whereby the participants' toenails would be cut and corns and calluses shaved 3. Instructing competent family members in the proper methods of cutting toenails and using commercial foot care products. 4. Recommending that the participants soak their feet for 10 minutes before cutting their toenails using safe toenail clippers Correct Response: 1 64. The occurrence of tuberculosis in the older adult is significantly increased among individuals who 1. are physically inactive. 2. are cigarette smokers. 3. have received the Bacile Calmette Gurin (BCG) vaccine. 4. reside in institutions. Correct Response: 4 65. Which type of fracture has the highest morbidity and mortality in the older adult? 1. Ankle

2. Hip 3. Shoulder 4. Vertebral Correct Response: 2 66. The primary reason for establishing quality improvement committees at long-term care facilities is to: 1. facilitate staff participation. 2. initiate changes based on interdisciplinary exchange. 3. monitor and record incidents, accidents, and injuries. 4. provide quality care based on measurable data. Correct Response: 4 67. An accrediting body evaluates a nursing home by monitoring the number of residents who developed pressure ulcers and urinary tract infections. What type of audit is being conducted? 1. Outcomes 2. Process 3. Prospective 4. Structure Correct Response: 1 68. A gerontological nurse is creating a staff development program for a unit. To assess the staff's learning needs, the nurse's best approach is to ask staff members: 1. How do you want to learn new material? 2. What do you need to know to do your job better? 3. What do you think others need to learn?

4. What do you think others want you to learn? Correct Response: 2 69. A comprehensive staff development program in a longterm care facility is based on the: 1. availability of educational resources. 2. nursing director's perceptions of staff learning needs. 3. philosophy, goals, and objectives of the organization. 4. recommendations of the ombudsman. Correct Response: 3 70. A gerontological charge nurse delegates the administration of a nasogastric tube feeding to a licensed practical nurse (LPN). Which statement about this situation is most accurate? 1. The charge nurse is responsible for delegated care. 2. The charge nurse should implement the care and not delegate the task. 3. The LPN is accountable for his or her own actions. 4. The LPN should respectfully refuse to initiate this care. Correct Response: 1 71. Which profession first developed standards of gerontological care and provided a certification mechanism to ensure expertise? 1. Medicine 2. Nursing 3. Physical therapy 4. Social work Correct Response: 2

72. The gerontological nurses in a teaching nursing home are informed of a research project to obtain voiding cystometrograms on all residents. The gerontological nurses' responsibility is to ensure that the: 1. non-English speaking residents receive a complete explanation of the study in their native languages. 2. nursing home administration know that the nurses do not support such invasive studies on the residents. 3. research team is well staffed so that the nursing home staff can provide care as usual. 4. residents will be properly medicated in the examining room where the tests will be conducted. Correct Response: 1 73. Which is an accurate statement about Medicare? 1. Medicare is a health insurance program with carefully described benefits that may restrict the length of hospital stays. 2. Medicare patients can expect to receive routine household and attendant care under Part A of their Medicare insurance. 3. Medicare pays for the majority of nursing home care for persons who are 65 years of age or older. 4. Persons covered by Medicare must pay once-in-a-lifetime deductibles on both the physician and hospital portions of the program. Correct Response: 1 74. Nursing facilities that receive federal funds must complete for all residents a:

1. minimum data set and care plan within seven days. 2. resident assessment instrument and care plan on admission. 3. resident assessment instrument and care plan within 14 days. 4. resident assessment instrument within 14 days and a care plan within 21 days. Correct Response: 4 75. The primary function of an ombudsman is to: 1. act as a clearinghouse for complaints and problems. 2. initiate complaints about the facility's operations. 3. question the facility administrator and the director of nursing. 4. welcome and interview patients and their families. Correct Response: 1

The gerontology nurse practitioner should rule out (R/O) which symptom before the diagnosis of dementia is made?
History of seizures (convulsions) History of head injury History of schizophrenia History of delirium Because cognition impairment caused by delirium might be reversible, delirium must be ruled out when diagnosing dementia.

The definitive medical diagnosis of AD is confirmed by:


Histopathological findings. History and physical examination. Neuropsychological testing. Combing estimates of genetic and environmental risk factors. The definitive diagnosis of AD is provided only after histopathological confirmation at autopsy. However, clinical diagnosis can be made from history, physical examination, and neuropsychological testing.

The majority of cases of Alzheimer's disease (AD) result from complex interactions between genetic and environmental factors. Studies have shown that the major risk factor for developing AD is the 4 allele of the apolipoprotein (APOE) on chromosome 19.

Which of the following statements is accurate pertaining to frontotemporal dementia? Frontotemporal dementia: (Select all that apply.)
Includes Lewy bodies, and is diagnosed on the basis of personality changes. Is an abrupt-onset dementia with abnormal reflexes and emotional ability.

Includes Pick's disease, and is diagnosed on the basis of personality changes and the presence frontal brain atrophy. Includes Lewy bodies, Pick's inside nerve cells, and behavioral disinhibition. Frontotemporal dementia, which includes Pick's disease, is diagnosed on the basis of personality changes and the presence of frontal brain atrophy.

Carbenicillin Indanyl sodium (Geocillin) Carboplatin (Paraplatin) Carisoprodol (Rela) The pathology of PD is related to the loss of dopaminergic cells situated deep in the midbrain in the substantia nigra. Dopamine is necessary for neurotransmission of nerve terminals, which is involved in the initiation of movement. Levodopa is the metabolic precursor of dopamine that provides symptomatic relief of some of PD symptoms, especially bradykinesia. Levodopa is combined with Carbidopa, which is a peripheral dopa-carboxylase inhibitor.

Parkinsonian disease (PD) is referred to as an extrapyramidal syndrome because it manifests which of the following clinical symptoms?
Tremor and bradykinesia Diarrhea and sweating Constipation and hypotension Somnolence and poor gait Parkinsonian disease (PD) has variable combinations of signs and symptoms that include tremor, rigidity, bradykinesia, and disturbances in gait.

Nurses who administer Sinemet as prescribed for the elderly should incorporate timing of the administration of Sinemet into the patient's care plan and:
Give Sinemet 30 minutes after meals. Give Sinemet 30 minutes before meals. Give Sinemet one hour before meals. Give Sinemet with meals, followed by a glass of water. Sinemet should be taken on an empty stomach. The nurse should incorporate timing of the administration of Sinemet into the patient's plan of care. Both the patient and caregiver should be taught that Sinemet should be taken one hour before or two hours after a meal.

Which of the following drugs would be ordered for persons with Parkinsonian disease?
Carbidopa/Levodopa (Sinemet)

The pathology of a stroke typically is not caused by which of the following?


Hemorrhage into the brain An embolus or thrombus that occludes an

artery Rupture of an extracerebral artery, causing subarachnoid hemorrhage Damage to the cerebrum and myelin sheath The pathology of a stroke typically is caused by hemorrhage into the brain, an embolus or thrombus that occludes an artery, or rupture of an extracerebral artery, causing subarachnoid hemorrhage.

Measuring the patient's height and weight accurately. There are several nursing interventions when caring for a patient with a history of seizures. It is the nurse's responsibility to obtain an accurate patient history, including age of seizure onset and frequency of attacks. It is important for the nurse to be aware of the medication dose, because many patients are given a starting dose, which might need to be titrated.

Which prescription most likely would be ordered if not contraindicated for acute ischemic stroke?

The most important responsibility of the nurse when a patient has a seizure is to:
Maintain the head-tilt-chin lift method. Provide oxygen immediately. Prevent injury of the patient. Prevent status epilepticus. If a patient has a seizure, the most important responsibility of the nurse is to prevent injury to the patient. Oxygen should always be available, and the head-tilt-chin method can be used to obtain an airway.

Packed red blood cells (PRBC), followed by furosemide (Lasix) 25% albumin followed by 0.9% sodium chloride Recombinant-tissue plasminogen activator (rtPA) Heparin infusion alternating with coumadin (Warfarin)

If thrombolytic therapy is not contraindicated, then recombinant tissue plasminogen activator (rt-PA) often will be administered within three hours to treat the acute ischemic stroke. However, not all patients treated with rt-PA therapy will have positive outcomes.

Nursing interventions for persons with history of seizure would include: (Select all that apply.)
Obtaining an accurate patient history. Knowledge of the frequency of attacks. Being aware of the medication dosage and frequency.

Patient-family education in regards to seizures would include: (Select all that apply.)
Providing patient and family with audiovisual aids for ongoing review. Teaching how to identify and avoid

precipitating factors for seizures. Stressing the need to take medication as prescribed. Teaching what to do in the event a seizure occurs.

Which of the following are aged-related changes that affect gastrointestinal function? (Select all that apply.)

Small, frequent feedings requires less release of hydrochloric acid. Remaining in an upright position for 30 minutes after meals prevents reflux into the esophagus.

Decline in sense of taste and smell, decrease in salivary secretion Diminished capacity of the gastric mucosa to resist such factors as Helicobacter pylori Achlorhydria or insufficient hydrochloric acid in the stomach Decreases in intestinal absorption, motility, and blood flow

Persons with GERD should be referred to a primary care provider for a thorough cardiac evaluation to rule out cardiac disease. The most frequently used diagnostic test is barium swallow. Upper endoscopy is the best method to assess mucosal injury. Acid perfusion tests usually are not necessary, and require the placement of an esophageal probe above the esophageal sphincter to collect esophageal contents.

Bed-bound patients should not be fed in the semi-reclined position or fed thin liquids that might quickly drain into the esophagus before the swallow reflex is triggered. A heavy caseload of clients might cause healthcare providers to rush-feed patients, resulting in reflux of nutrients and gastric acid.

Stroke, especially in the midbrain and anterior cortical areas, is the most common cause of dysphagia in the older person. Parkinson's disease and Alzheimer's disease can cause dysphagia by inhibiting movements of the tongue, pharynx, or upper esophagus. Crohn's and ulcerative colitis affect the G.I. mucosa and submucosa, and do not usually cause esophageal reflux or aspiration.

Preparation of the videofluoroscope radiographic evaluation includes drinking of a radio opaque solution, which is a chalk

mixture. The chalk mixture is constipating, and could cause fecal impaction.

Which of the following statements are accurate as they relate to medications used to manage GERD? (Select all that apply.)
Magnesium-containing antacids can cause diarrhea. Aluminum-containing antacids can cause constipation. Cimetidine (Tagamet HB) causes osteomalacia and hypophosphatemia. Misoprostol's (Cytotec) major side effect is G.I. bleeding. Magnesium-containing antacids can cause diarrhea, and should be used with caution in older persons with renal dysfunction. Aluminum-containing antacids can cause constipation, osteomalacia, and hypophosphatemia. Cimetidine has the greatest chance for adverse reactions, including erectile dysfunction, Gynecomastia, and confusion. Misoprostol's major side effects are diarrhea and abdominal pain.

Older patients with GERD and Barrett's esophagus require aggressive treatment with proton pump inhibitors and regular endoscopic examination. If the esophageal erosion does not reverse with treatment, the surgery of choice is Nissen's fundoplication, and involves closing any hiatal hernia and restoring an antireflux barrier.

Which of the following types of gastritis is associated with Helicobacter pylori and duodenal ulcers?
Erosive (hemorrhagic) gastritis Fundic gland gastritis (type A) Antral gland gastritis (type B) Aspiring-induced gastric ulcer Erosive (hemorrhagic) gastritis can be caused by ingestion of substances that irritate the gastric mucosa. Fundic gland gastritis (type A) is associated with diffuse severe mucosal atrophy and the presence of pernicious anemia. Antral gland gastritis (type B) is the most common form of gastritis, and is associated with Helicobacter pylori and duodenal ulcers.

The surgical procedure of choice for older patients with GERD and Barrett's esophagus that is not reversible with medical management is:
Esophagectomy. Total laryngectomy. Nissen's fundoplication. Labyrinthectomy.

Which of the following statements is accurate about patients diagnosed with Zollinger-Ellison syndrome?

Peptic ulcer occurs in 95% of patients diagnosed with Zollinger-Ellison syndrome. Zollinger-Ellison syndrome is characterized by gastric hyposecretion, because of dysfunction of the parietal cells. Patients with Zollinger-Ellison syndrome with persistent symptoms that do not respond to drug treatment should be referred to a radiologist.

Treatment of choice for Zollinger-Ellison syndrome is a cholecystectomy. Peptic ulcers occur in 95% of patients with Zollinger-Ellison syndrome. Treatment might include tumor removal and surgical resection for older persons without surgical risk. Persons with Zollinger-Ellison syndrome have gastric hypersecretion, and are referred to a gastroenterologist if symptoms persist.

In planning care for a patient with ulcerative colitis, the nurse should anticipate which of the following diagnostic procedures?
Sigmoidoscopy Colonoscopy Rectal mucosa biopsy

Which of these agents is a major contributing factor in the promotion of peptic ulcer disorder?
Candida albicans Staphylococcus infection Streptococcus infection Helicobacter pylori infection Recurrence of peptic ulcers is related to Helicobacter pylori, use of NSAIDs, smoking, and continued acid hypersecretion.

All of the above Diagnosis of ulcerative colitis is confirmed with the use of sigmoidoscopy, colonoscopy, and rectal mucosa biopsy.

The most common surgical procedures for patients with ulcerative colitis are:
Subtotal colectomy and ileostomy. Colostomy and ileo-conduit. Laparoscopic gastrectomy. Segmental resection or colostomy.

Which of the following is not an indication of diverticulitis?


Fever Leukocytosis Pain Diarrhea Diverticulitis is an infection from colonic diverticula. Fever, leukocytosis, pain, and/or abdominal tenderness might be indicators of diverticulitis.

Surgery might be necessary for functional older patients with acute disease when drug therapy fails and when multiple precancerous lesions are detected. The most common surgical procedures are subtotal colectomy and ileostomy.

Older patients with longstanding or severe Crohn's disease can exhibit which of the following?
Hyperalbuminemia Hypoalbuminemia

Decreased sedimentation rate Nausea and vomiting

Age-related changes that affect the hematologic system include: (Select all that apply.) Correct Answers: Bone marrow in the long bones decline. Lymphocyte function, especially cellular immunity, decreases. Rationale: At about age 70, the amount of bone marrow in the long bones declines steadily; the number of stem cells in the marrow decreases; and lymphocyte function, especially cellular immunity, declines. 2. Anemia or insufficient hemoglobin content is common in older persons. The client's body compensates for the deficiency by: Rationale: All anemias result in a loss of oxygen-carrying capacity of the blood, and produce generalized hypoxia. The body tries to compensate by raising the heart and respiratory rates, shunting blood to vital organs away from the skin, and increasing blood viscosity in order to supply oxygen to hypoxic tissues. 3. Which of the following statements accurately describes normocytic anemia? Normocytic anemia

usually is caused by: Your Answer: Concurrent chronic illness, such as chronic heart disease. Rationale: Normocytic anemia usually is caused by concurrent chronic illness such as chronic heart, respiratory, or renal disease or malignancy. Hemolytic anemia also is a normocytic anemia. 4. The usual treatment for iron-deficiency anemia includes: Your Answer: Non-enteric-coated ferrous sulfate. Rationale: Once the cause of iron deficiency has been identified, oral iron therapy is the preferred treatment. The usual treatment is ferrous sulfate 325 mg p.o. daily. Enteric-coated and sustained release formulations should be avoided, as they are poorly absorbed. ________________________________________ 5. Erythropoietin sometimes is administered subcutaneously to treat which of the following? (Select all that apply.) Correct Answer: Clients with anemia and fatigue related to non-myeloid cancers Rationale: Cancer in the late stages nearly always is accompanied by anemia of chronic disease. Erythropoietin, which stimulates red blood cell production, is injected subcutaneously, and

sometimes is used to treat anemia and fatigue in some older clients with non-myeloid cancers. 6. The most common cause of macrocytic anemia in the older person is B12 or folate deficiency. Failure to absorb vitamin B12 from the G.I. tract is called: Correct Answer: Pernicious anemia.

Rationale: Failure to absorb vitamin B12 from the G.I. tract is called pernicious anemia, and results from a lack of intrinsic factor. About 12% of older people have low serum vitamin B12 levels. 7. An elderly woman is seen at the outclient clinic by a nurse practitioner for a routine hematologic evaluation. Which of the following findings would alert the nurse practitioner of the need for a pending diagnosis of hemolytic anemia? Correct Answer: Ongoing use of cephalosporin, quinidine, triamterene, and aspirin Rationale: Drugs capable of causing hemolytic anemia include ibuprofen, L-dopa, penicillin, cephalosporins, tetracycline, acetaminophen, aspirin, erythromycin, hydralazine, insulin, isoniazid, procainamide, quinidine, rifampin, streptomycin, sulfonamides, methadone, hydrochlorothiazide, triamterene, and phenacetin. 8. The nurse has provided nutritional teaching on foods high in folate to a client with folate

deficiency related to malabsorption syndromes and poor nutrition. Which of the following foods, if chosen by the client, indicates that the client understands the teaching? Correct Answer: Liver and dark green leafy vegetables Rationale: Foods high in folate are liver, orange juice, cereals, whole grains, beans, nuts, and dark leafy vegetables like spinach. 9. Because older persons can have severe anemia for a long period of time without detection, when diagnosed, quick reversal is warranted. Which of the following orders most likely would be prescribed at this time? Your Answer: Ferrous sulfate 325 mg orally three times a day Correct Answer: Packed red blood cells followed by oral furosemide (Lasix) Rationale: Older persons might have heart problem that are compounded by severe anemia. The physician can prescribe blood transfusions to reverse the severity of the anemia, and a diuretic such as furosemide (Lasix) orally between units to prevent fluid overload and the development of congestive heart failure (CHF). 10. An elderly client is admitted to the hospital Emergency Department (ED) with complaints of headache, visual disturbances, and burning pain, and erythema of the hands and feet. To accurately

diagnose thrombocytopenia, the physician most likely will order: Correct Answer: Bone marrow aspiration.

Rationale: Thrombocytopenia is characterized by an increased number of circulating platelets in the blood. Accurate diagnosis requires bone marrow aspiration. Allogenic bone marrow transplantation is prescribed for younger persons with myelofibrosis. A splenectomy may be prescribed for persons with myelofibrosis. 11. When caring for a client with a diagnosis of thrombocytopenia, the nurse should plan to: Your Answer: Avoid invasive procedures. Rationale: Thrombocytopenia is characterized by an increased number of circulating platelets in the blood. Older persons with thrombocytopenia are at significantly increased risk of thrombosis, and careful monitoring of platelet levels and symptoms is indicated. 12. In formulating a nursing diagnosis of risk for infection for a client with chronic lymphoid leukemia (CLL), nursing measures should include: (Select all that apply.) Correct Answers: Placing the client in protective isolation. Limiting visitors who have colds and infections. Ensuring meticulous handwashing by all persons

coming in contact with the client. Rationale: Chronic lymphoid leukemia (CLL) is characterized by a proliferation and accumulation of small, abnormal mature lymphocytes in bone marrow, peripheral blood, and body tissues. Infections and fever are frequent complications of CLL. 13. Which question by the nurse would be beneficial in assessing the needs of a client with multiple myeloma? Correct Answer: "Describe your pain."

Your Answer: "It only hurts when I move. I don't need any pain medication." Rationale: Postoperative and chronic pain should be carefully assessed and treated to prevent older clients from the hazards of immobility. The older client who says, "It only hurts when I move. I don't need any pain medication," is at risk for immobility and DVT formation. Which of the following is the major mechanism for maintaining calcium balance in the body? Correct Answer: Bone remodeling Rationale: Bone accounts for roughly 99% of calcium in the body. Throughout life, new bone continually is deposited and reabsorbed in response to hormonal, dietary, and mechanical stimuli. When put together, these processes are called bone remodeling. Body alignment does not directly affect calcium balance. Exercise, particularly weight-bearing exercise, is important for bone health.

Rationale: Multiple myeloma is a malignancy that results from the overproduction and accumulation of immature plasma cells in the bone marrow. Bone pain in the lower back or ribs is the most common early symptom of multiple myeloma. 14. Which of the following manifestations would be directly associated with Hodgkin's disease? Correct Answer: Painless, enlarged lymph nodes

Rationale: Hodgkin's disease usually presents as painless enlarged lymph nodes. The diagnosis is made by lymph node biopsy. 15. Which of the following statements made by the older client indicates risk for immobility and deep vein thrombosis (DVT) formation?

2. Osteoporosis can be defined as: Your Answer: Loss of bone density. Rationale: Osteoporosis is loss of bone density. Specific causes in the elderly include Vitamin D deficiencies and the use of glucocorticoid

drugs. Phagocytic activity is not a contributing factor here. 3. Which of the following risk factors are directly related to common musculoskeletal problems in older persons? (Select all that apply.) Correct Answers: Phosphate depletion Vertebral collapse Alteration in vitamin D metabolism Osteoid bone is produced in excess of mineralization. 4. Which of the following nursing diagnoses would be assigned priority for the older person with common musculoskeletal problems? Correct Answer: Mobility impairment, physical: Related to pain or discomfort Rationale: Musculoskeletal status - including coordination, gait, muscle size and strength, muscle tone, range of motion, and functional mobility - will cause altered mobility or impairment in mobility for patients with common musculoskeletal problems. 5. Which of the following patients with musculoskeletal problems should the nurse observe first? Correct Answer: A client who has just returned from the operating room (OR)

Rationale: Immediate interventions such as airway management and vital signs are priority. The patient with the fractured ankle is requesting pain medication, but the type of pain is not indicated. The client with the wet dressing is likely the lowestpriority client. 6. The physician prescribes alendronate (Fosamax) and risedronate (Actonel) for a 65-yearold female for which of the following purposes? Correct Answer: Inhibiting osteoclastic activity, and decreasing the incidence of vertebral and nonvertebral fractures Rationale: Fosamax and Actonel are potent drugs that inhibit osteoclastic activity and help to decrease the incidence of vertebral and nonvertebral fractures in post-menopausal women. The actions of these drugs do not include preventing thrombus formation, binding bile acids, or acting as an anti-inflammatory agent. 7. Which of the following nursing implications are important when Fosamax and Actonel are prescribed for an older patient? (Select all that apply.) Correct Answers: Take these drugs on an empty stomach, first thing in the morning, with water. Remain upright for 30 minutes and do not eat or drink anything else for 30 minutes.

Rationale: Fosamax causes esophagitis if there is prolonged contact with the esophageal mucosa, or if it does not pass through the esophagus completely. Therefore, Fosamax must be taken with a sufficient amount of water. To promote complete passage of the drug, it should be taken in an upright position, to prevent delay of the drug through the esophagus. 8. A patient is diagnosed with osteomalacia. The physician prescribes vitamin D replacement 50,000 U/day for two weeks, to be followed by a daily dose of 800 U/day. The primary goal for this pharmacological treatment is that: Your Answer: The patient will achieve effective remineralization of bone structure. Rationale: The goal of pharmacological treatment for osteomalacia is to remineralize the bone. All older persons with osteomalacia need to have adequate vitamin D intake to help alleviate calcium deficiency. 9. Which of the following is a major factor in the susceptibility and severity of rheumatoid arthritis (RA)? Correct Answer: Genetic predisposition Rationale: The cause of RA is unknown, but it most likely is due to a variety of unknown environmental factors, including infectious agents and chemical exposures, all triggering an autoimmune response. However, a person's genetic

predisposition is a major factor in both the susceptibility and the severity of RA. 10. The focus of treatment for rheumatoid arthritis is related directly to joint damage that is caused by: Correct Answer: Pannus.

Rationale: Pannus is vascular granulation tissue composed of inflammatory cells that erodes articular cartilage and eventually destroys bones. A vasculitis usually indicates blood vessel involvement. Sjgrens syndrome is due to inflammatory cells and immune complexes obstructing secretory ducts and glands. Felty's syndrome is characterized by rheumatoid arthritis, hepatosplenomegaly, and leukopenia. 11. is: The definitive finding for persons with gout Urate crystals in synovial fluid.

Correct Answer:

Rationale: Gout results from the deposit of urate crystals in a peripheral joint. The definitive finding for a diagnosis of gout is urate crystals in the synovial fluid, usually via an arthrocentesis. Hyperuricemia is excess uric acid in the blood. Diffuse periarticular erythema is not definitive for gout. 12. A patient with pseudogout most likely would

complain of pain in what specific area of the body? Your Answer: Knee Rationale: The knee joint is the most common joint affected for most people. Other joints that might be affected include the shoulder, hip, and elbow. 13. Which of the following statements is accurate about hip fractures? Correct Answer: Extracapsular fractures cause acute blood loss from the vascular cancellous bone surfaces, but rarely cause avascular necrosis. Rationale: Extracapsular fractures cause acute blood loss from the vascular cancellous bone surfaces. 14. Which of these findings correlates best with a diagnosis of Paget's disease? Correct Answer: Complaints of bone pain Rationale: Bone pain is the most frequently reported symptom, described as deep and aching. Hyperuricemia is present, and the skin usually is flushed and warm. Nausea and vomiting are a nonspecific presentation. 15. Principles related to nursing management of older persons with arthritis include: (Select all that apply.)

Correct Answers: Promoting a diet with adequate calcium and vitamin D Encouraging weight-bearing exercise Emphasizing the importance of smoking cessation Rationale: Osteoarthritis is the most common form of arthritis in the United States, more women are affected than are men? Recent studies have determined that aging alone does not cause the disease; therefore, any woman, by the age of 40, might show evidence of primary osteoarthritis on xray. Which of the following persons would most likely be diagnosed with diabetes mellitus? A 44-year-old: Correct Answer: African-American woman. Rationale: Age-specific prevalence of diagnosed diabetes mellitus (DM) is higher for African-Americans and Hispanics than for Caucasians. Among those younger than 75, black women had the highest incidence. 2. Which of the following factors are risks for the development of diabetes mellitus? (Select all that apply.) Correct Answers: Age over 45 years Overweight with a waist/hip ratio >1 Maintaining a sedentary lifestyle

Rationale: Aging results in reduced ability of beta cells to respond with insulin effectively. Overweight with waist/hip ratio increase is part of the metabolic syndrome of DM II. There is an increase in atherosclerosis with DM due to the metabolic syndrome and sedentary lifestyle. 3. Which laboratory test should a nurse anticipate a physician would order when an older person is identified as high-risk for diabetes mellitus? (Select all that apply.) Correct Answers: Fasting Plasma Glucose (FPG)

Nursing Process: Assessment Client Need: Health Promotion and Maintenance Taxonomic Level: Application ________________________________________ 5. When an older adult is admitted to the hospital with a diagnosis of diabetes mellitus and complaints of rapid-onset weight loss, elevated blood glucose levels, and polyphagia, the gerontology nurse should anticipate which of the following secondary medical diagnoses? Your Answer: Pituitary tumor Correct Answer: Pancreatic tumor Rationale: The onset of hyperglycemia in the older adult can occur more slowly. When the older adult reports rapid-onset weight loss, elevated blood glucose levels, and polyphagia, the healthcare provider should consider pancreatic tumor. Nursing Process: Assessment Client Need: Physiologic Integrity, Reduction of Risk Potential Taxonomic Level: Evaluation ________________________________________ 6. The principal goals of therapy for older patients who have poor glycemic control are: Your Answer: Improving self-care through education. Correct Answer: All of the above. Rationale: The principal goals of therapy for older persons with diabetes mellitus and poor

Two-hour Oral Glucose Tolerance Test (OGTT) Rationale: When an older person is identified as high-risk for diabetes, appropriate testing would include FPG and OGTT. A FPG greater than 140 mg/dL usually indicates diabetes. The OGTT is to determine how the body responds to the ingestion of carbohydrates in a meal. HbA1C evaluates longterm glucose control. A finger stick glucose three times daily spot-checks blood glucose levels. 4. Of which of the following symptoms might an older woman with diabetes mellitus complain? Your Answer: Pain intolerance Correct Answer: Perineal itching Rationale: Older women might complain of perineal itching due to vaginal candidiasis.

glycemic control are enhancing quality of life, decreasing the chance of complications, improving self-care through education, and maintaining or improving general health status. Nursing Process: Planning Client Need: Safe, Effective Care Environment Taxonomic Level: Comprehension ________________________________________ 7. Which of the following is accurate pertaining to physical exercise and type 1 diabetes mellitus? Your Answer: Strenuous exercise is beneficial when the blood glucose is high. Correct Answer: Physical exercise can slow the progression of diabetes mellitus. Rationale: Physical exercise slows the progression of diabetes mellitus, because exercise has beneficial effects on carbohydrate metabolism and insulin sensitivity. Strenuous exercise can cause retinal damage, and can cause hypoglycemia. Insulin and foods both must be adjusted to allow safe participation in exercise. Nursing Process: Application Client Need: Physiologic Integrity Taxonomic Level: Analysis ________________________________________ 8. A diabetic patient experiencing a reaction of alternating periods of nocturnal hypoglycemia and hyperglycemia might be manifesting which of the following? Your Answer: Brittle diabetes

Correct Answer:

Somogyi phenomenon

Rationale: Somogyi phenomenon manifests itself with nocturnal hypoglycemia, followed by a marked increase in glucose and increase in ketones. Nursing Process: Analysis Client Need: Physiologic Integrity Taxonomic Level: Analysis ________________________________________ 9. The primary purpose for sulfonylureas, such as longacting glyburide (Micronase), is to: Your Answer: Improve insulin sensitivity and decrease hyperglycemia. Correct Answer: Stimulate the beta cells of the pancreas to secrete insulin. Rationale: Sulfonylureas such as glyburide are used only with patients who have some remaining pancreatic-beta cell function. These drugs stimulate insulin secretion, which reduces liver glucose output and increases cell uptake of glucose, enhancing the number of and sensitivity of cell receptor sites for interaction with insulin. Nursing Process: Analysis Client Need: Physiologic Integrity, Pharmacologic Therapy Taxonomic Level: Knowledge ________________________________________ 10. One of the benefits of Glargine (Lantus) insulin is its ability to:

Your Answer: Simplify the dosing and better control blood glucose levels during the day. Correct Answer: Release insulin evenly throughout the day and control basal glucose levels. Rationale: Glargine (Lantus) insulin is designed to release insulin evenly throughout the day and control basal glucose levels. Nursing Process: Evaluation Client Need: Physiologic Integrity, Pharmacologic Therapy Taxonomic Level: Evaluation ________________________________________ 11. A frail elderly patient with a diagnosis of type 2 diabetes mellitus has been ill with pneumonia. The client's intake has been very poor, and she is admitted to the hospital for observation and management as needed. What is the most likely problem with this patient? Your Answer: Diabetic ketoacidosis is occurring. Correct Answer: Hyperglycemic hyperosmolar non-ketotic coma Rationale: Illness, especially with the frail elderly patient whose appetite is poor, can result in dehydration and HHNC. Insulin resistance usually is indicated by a daily insulin requirement of 200 units or more. Diabetic ketoacidosis, an acute metabolic condition, usually is caused by absent or markedly decreased amounts of insulin. Nursing Process: Analysis

Client Need: Physiologic Integrity Taxonomic Level: Analysis ________________________________________ 12. Which of the following diagnostic studies is done to determine the size and composition of the thyroid gland? Your Answer: Thyroid scan with RAI 123I Correct Answer: Ultrasonography Rationale: Although thyroid scans frequently are done to evaluate the thyroid gland, I 123 is used to destroy overactive thyroid cells such as are seen in thyroid cancer. Ultrasonography can be used early in the evaluation process to rule out Graves' disease, nodular goiter, or other thyroid dysfunction. Nursing Process: Assessment Client Need: Physiologic Integrity, Reduction of Risk Potential Taxonomic Level: Knowledge ________________________________________ 13. Which of the following is a drug of choice for hyperthyroidism (Graves' disease)? Your Answer: Digoxin (Lanoxin) Correct Answer: Propylthiouracil (PTU) Rationale: Propylthiouracil (PTU) initially is given in divided doses, and functions to block thyroid hormone synthesis. Nursing Process: Assessment Client Need: Physiologic Integrity, Pharmacologic Therapy

Taxonomic Level: Knowledge ________________________________________ 14. The nurse is caring for a patient with a diagnosis of hypothyroidism. Which nursing diagnosis should the nurse most seriously consider when analyzing the needs of the patient? Your Answer: Diarrhea related to increased peristalsis Correct Answer: Hypothermia related to slowed metabolic rate Rationale: Thyroid hormone deficiency results in reduction in the metabolic rate, resulting in hypothermia, and does predispose the older adult to a host of other health-related issues. One quarter of affected elderly experience constipation. Nursing Process: Planning Client Need: Physiologic Integrity, Reduction of Risk Potential Taxonomic Level: Analysis ________________________________________ 15. Which of the following medications are most likely to cause hypothyroidism? (Select all that apply.) Your Answer: Acetylsalicylic acid (aspirin) Correct Answers: Acetylsalicylic acid (aspirin) Furosemide (Lasix) Rifampin (Rifadin) Nursing Process: Assessment

Client Need: Physiologic Integrity, Pharmacologic Therapy Taxonomic Level: Knowledge Which of the following are least likely to be early signs of cardiac problems in older persons? (Select all that apply.) Your Answer: Mental status changes Correct Answers: Mental status changes Agitation Frequent falls Rationale: Many cardiovascular functions are complicated in that they involve many other systems. Mental status changes, agitation, and falls can be early signs of cardiac problems in the older person. Changes in function in the GI system are not typical signs of a cardiac problem. Integrated Process: Nursing Process; Evaluation Cognitive Level: Analysis NCLEX-RN Test Plan: Physiological Integrity; Reduction of Risk ________________________________________ 2. A patient has been diagnosed with Right-Sided Congestive Heart Failure, and is confused about return of deoxygenated blood from the tissue. To clarify the confusion, which chamber of the heart receives blood from systemic circulation? Your Answer: Right atrium

Rationale: The right atrium is a thin-walled structure that receives deoxygenated blood from all the peripheral tissues by way of the superior and inferior vena cava and from the heart muscle by way of the coronary sinus. Integrated Process: Teaching Learning Cognitive Level: Knowledge NCLEX-RN Test Plan: Health Promotion and Maintenance ________________________________________ 3. It is important that the nurse be knowledgeable about cardiac output in order to: Your Answer: Determine the electrical activity of the myocardium. Correct Answer: Evaluate blood flow to peripheral tissues. Rationale: Blood flow to the tissues is measured clinically as the cardiac output, and assists to predict tissue perfusion. Electrical activity is evaluated more effectively by EKG. While the cardiac output is important for perfusion and oxygenation of tissues, the oxygen saturation would provide more valuable information. Nutritional changes would be targeted to sodium and would depend on symptoms of disease. Integrated Process: Nursing Process; Planning Cognitive Level: Evaluation NCLEX-RN Test Plan: Health Promotion and Maintenance ________________________________________ 4.

Nurses can best help older clients prevent hypertension by teaching: Your Answer: Low-fat, low-cholesterol diets. Correct Answer: How to maintain a normal blood pressure. Rationale: Hypertension is a major risk factor for other cardiovascular conditions. In persons older than 50, systolic blood pressure greater than 140 mm Hg is a much more important cardiovascular disease risk factor than is diastolic blood pressure. The risk of cardiovascular disease, beginning at 115/75 mm Hg, doubles with each increment of 20/10 mm Hg. Answers 1, 2, and 3 are important elements to include in education of a patient with blood pressure elevation, and are included in the correct answer. Integrated Process: Nursing Process; Implementation Cognitive Level: Evaluation NCLEX-RN Test Plan: Health Maintenance and Promotion ________________________________________ 5. Modification of lifestyle behaviors to help manage hypertension does not include which of the following? (Select all that apply.) Your Answer: The DASH diet Correct Answer: Alcohol intake with meals Rationale: Weight loss of even 10 pounds; fruits, vegetables, and whole grains; the DASH diet; and a daily exercise regimen will help reduce high

blood pressure. Alcohol intake with meals should be reduced to help manage high blood pressure. Integrated Process: Nursing Process; Planning Cognitive Level: Synthesis NCLEX-RN Test Plan: Health Maintenance and Promotion ________________________________________ 6. A patient receiving the drug simvastatin (Zocor) should be taught this medication helps to prevent coronary heart disease by: Your Answer: Increasing triglycerides. Correct Answer: Controlling lower-density lipoprotein. Rationale: The Heart Protective Study has also shown that controlling low-density lipoprotein (LDL) with simvastatin (Zocor) assists in the prevention of coronary heart disease by raising HDL. Responses 1, 3, and 4 are incorrect. Integrated Process: Teaching Learning Cognitive Level: Analysis NCLEX-RN Test Plan: Physiological Integrity: Pharmacological and Parenteral Therapies ________________________________________ 7. Which of the following are not direct causes of chronic ischemic pain? (Select all that apply.) Your Answer: Acid reflux Correct Answers: Aortic stenosis Acid reflux

Pulmonary embolus Rationale: Herpes zoster (shingles) manifests as a vesicular rash along a dermatome, not chronic ischemic pain. Integrated Process: Nursing Process; Evaluation Cognitive Level: Evaluation NCLEX-RN Test Plan: Physiological Integrity; Physiological Adaptation ________________________________________ 8. Older clients experiencing anginal pain with complaints of fatigue or weakness usually are medicated with which of the following types of medication? Your Answer: Sublingual nitroglycerin Rationale: Angina frequently is managed with sublingual nitroglycerin, which causes vasodilation and increases blood flow to the coronary arteries. Cardiac glycosides are used to treat heart failure, and morphine is used to treat myocardial infarction. The HMG-CoA reductase inhibitors are used for patients with type 2 diabetes mellitus. Integrated Process: Nursing Process; Analysis Cognitive Level: Analysis NCLEX-RN Test Plan: Physiological Integrity; Pharmacological Therapy ________________________________________ 9. Which of the following diagnostic studies most likely would confirm a myocardial infarction? Your Answer: Creatinine kinase (CK)

Correct Answer:

Troponin T levels

Rationale: CK-MB elevates 4-6 hours after tissue necrosis. Troponin levels rise 6-8 hours after the infarct (tissue necrosis) but also can occur with other types of tissue damage. Myoglobin also elevates, but to a lesser degree. WBC levels elevate with an inflammatory response. Troponin levels are more elevated than are the other cardiac enzymes, are more specific to cardiac tissue, and rise 6-8 hours after the infarct (tissue necrosis). Integrated Process: Nursing Process: Analysis Cognitive Level: Analysis NCLEX-RN Test Plan: Physiological Integrity; Reduction of Risk Potential ________________________________________ 10. A client with post-myocardial infarction develops acute bacterial pericarditis. Which of the following medications would the physician most likely prescribe as the primary drug? Your Answer: Acetaminophen (Tylenol) Correct Answer: Ticarcillin disodium (Ticar) Rationale: Acute bacterial pericarditis is a complication that can occur post-myocardial infarction. Acute bacterial pericarditis usually requires antibiotics. NSAIDs usually are prescribed to relieve pain from the inflammatory process. If the NSAIDs do not relieve pain within 48 to 96 hours, corticosteroids are ordered. There is no mention of pain in the stem of the question. Trisoralen is used to repigment skin for persons with vitiligo.

Integrated Process: Nursing Process; Assessment Cognitive Level: Analysis NCLEX-RN: Physiological Integrity; Pharmacological Therapy ________________________________________ 11. Which of the following diagnostic tests is preferred for evaluating heart valve function? Your Answer: Echocardiogram Rationale: The echocardiogram is the preferred test to evaluate heart valves, because it allows the visualization of the valves as they open and close. A chest x-ray will determine the size of the heart, the duplex measures blood flow through major arteries, and an electrocardiogram identifies electrical activity. Integrated Process: Nursing Process; Analysis Cognitive Level: Analysis NCLEX-RN Test Plan: Physiological Integrity; Reduction of Risk Potential ________________________________________ 12. An elderly client is being monitored for evidence of congestive heart failure. To detect early signs of heart failure, the nurse would instruct the certified nursing attendant (CNA) to do which of the following during care of the patient? Your Answer: Observe electrocardiogram readings and report deviations to the nurse. Correct Answer: Accurately weigh the patient, and report and record the readings.

Rationale: Due to fluid accumulation, an expanded blood volume can result when the heart fails. Body weight is a sensitive indicator of water and sodium retention, which will manifest itself with edema, dyspnea - especially nocturnal - and pedal edema. Patients also should be instructed about the need to perform daily weights upon discharge to monitor body water. It is not within the role of the CNA to monitor ECG readings, and ambulation is not an assessment. Vital signs every 15 minute are not necessary for this level of patient care. Integrated Process: Communication and Documentation Cognitive Level: Planning NCLEX-RN Test Plan: Safe, Effective Care Environment ________________________________________ 13. Which of the following drug classifications should the nurse question if prescribed for a person with congested heart failure (CHF)? Your Answer: Beta-adrenergic blocker Correct Answer: Rosiglitazone (Avandia) Correct answer: Thiazolidinediones, like rosiglitazone (Avandia), are glucose-reducing drugs that are prescribed for persons with type 2 diabetes mellitus. ACE inhibitors, such as Lisinopril, are firstline drugs used to treat CHF. Propranolol (Inderal), a beta blocker, has remained one of the most widely used beta-blocking drugs. It blocks both beta1 and beta2 receptors in various organs, resulting in reduction of heart rate and the force of contraction,

and suppresses impulse conduction through the AV node, all of which slows the progression of the disease process. Carvedilol (Coreg) is another betaadrenergic blocker used to treat heart failure. Integrated Process: Nursing Process; Assessment Cognitive Level: Analysis NCLEX-RN Test Plan: Physiological Integrity; Pharmacological Therapy ________________________________________ 14. A common arrhythmia found in some older clients is chronic atrial fibrillation. Based on the nurse's knowledge of the disease pathology, which of the following prescriptions should the nurse expect to be ordered? Your Answer: Warfarin sodium (Coumadin) Rationale: Chronic atrial fibrillation places a patient at high risk for clot formation. Warfarin sodium frequently is ordered as an anti-coagulant. Aspirin will not prevent clots associated with atrial fibrillation. Zocor is used to lower LDL and increase HDL. Navelbine is an anti-neoplastic. Integrated Process: Nursing Process; Analysis Cognitive Level: Analysis NCLEX-RN Test Plan: Physiological Integrity; Pharmacological Therapy ________________________________________ 15. Which of the following assessments would be an important finding for a patient with arterial disease? Your Answer: Intermittent claudication with exercise

Rationale: Intermittent claudication is a common finding in persons with arterial disease, usually due to progression of atherosclerosis and alteration of tissue perfusion to the extremities. In venous disease, valves of the veins in the extremities become incompetent, resulting in higher pressures than normal in the veins. The pressure is transmitted to the capillaries of the lower extremities, resulting in thickening and non-pitting edema of tissues around the ankles. Prolonged thickening results in the red blood cells' being pressed outside the capillaries. The cells eventually break down, resulting in collection of hemosiderin deposits being collected in the area. Altered sensation to touch would be due to neuropathic changes commonly found with diabetes mellitus. Integrated Process: Nursing Process: Analysis Cognitive Level: Analysis NCLEX-RN Test Plan: Physiological Integrity; Reduction of Risk Potential Which of the following is a normal finding when assessing the respiratory system of an elderly client? Your Answer: A decreased anteroposterior diameter and increased alveolar surface Correct Answer: Decreased mobility of the thorax and increased chest wall stiffness Rationale: The normal aging process is accompanied by physiologic changes to the respiratory system such as stiffening of elastin and collagen tissues, increased alveolar diameter,

decreased alveolar surface, and increased chest wall stiffness. Bronchovesicular breath sounds are heard over major bronchi where fewer alveoli are located. Integrated Process: Nursing Process; Assessment Cognitive Level: Analysis NCLEX-RN Test Plan: Health Promotion and Maintenance ________________________________________ 2. An older client is receiving postural drainage treatments but is unable to expel the secretions. The client is confused, and having difficulty following instructions. The best response by the nurse would be to: Your Answer: Administer humidified oxygen. Correct Answer: Suction out the secretions. Rationale: Postural drainage loosens secretions, which patients usually expectorate when loosened. Since the client is confused, and cannot follow instructions, the nurse must suction the secretions to enhance effective breathing. Integrated Process: Nursing Process; Implementation Cognitive Level: Synthesis NCLEX-RN Test Plan: Physiological Integrity: Physiological Adaptation ________________________________________ 3. Chest percussion has been ordered for a client on bedrest with respiratory infections and increased secretions. The nurse should use which of the

following hand positions to administer chest percussion? Your Answer: Cupped-hand position Rationale: Cupped-hand position enhances vibration, which helps to loosen secretions, clear airway, and improve gas exchange. The flat, fisted, and flexed hand positions can cause pain and harm the patient. Integrated Process: Nursing Process; Implementation Cognitive Level: Analysis NCLEX-RN Test Plan: Physiological Integrity; Reduction of Risk Potential ________________________________________ 4. All of the following nursing diagnoses are important for a client with chronic pulmonary emphysema (COPD). Which would receive priority when planning nursing interventions? Your Answer: Activity intolerance Correct Answer: Impaired gas exchange Rationale: Physical assessment of older persons with COPD might find ineffective airway clearance, but the classic physiology alteration is the destruction of the alveoli with resulting complications. Therefore, impaired gas exchange always is present. Integrated Process: Nursing Process; Analysis Cognitive Level: Analysis NCLEX-RN Test Plan: Physiological Integrity; Adaptation

________________________________________ 5. The nurse is teaching a group of older clients about the functions of the respiratory tract as part of health promotion. Which statement made by patients would indicate a need for further teaching? Your Answer: "Parts of our respiratory system help us produce speech and better communication." Correct Answer: "The lungs keep our blood pressure in normal range, so we can breathe better." Rationale: The respiratory system performs several secondary functions, including maintenance of acid-base balance, production of speech, and maintenance of body water and heat balance. The heart and cardiovascular system are responsible for the blood pressure. Integrated Process: Teaching Learning Cognitive Level: Analysis NCLEX-RN Test Plan: Health Promotion and Maintenance ________________________________________ 6. Which of the following diagnostic studies is most helpful in determining the presence of asthma? Your Answer: Spirometry Correct Answer: Pulmonary function tests (PFTs) Rationale: Pulmonary function tests are the most reliable way to diagnose asthma and differentiate it from other illnesses like COPD. Large numbers of eosinophils indicate an allergic

component and a predictor of asthma. Spirometry is used to measure the volume of air expired in one second from maximum inspiration. Incentive spirometer is used for breathing exercises with patients who have respiratory problems; pre- and postop patients also benefit from the use of the incentive spirometer. Integrated Process: Nursing Process; Assessment Cognitive Level: Analysis NCLEX-RN Test Plan: Physiological Integrity; Reduction of Risk Potential ________________________________________ 7. Corticosteroid therapy is the most effective antiinflammatory treatment for asthma. The nurse administering oral corticosteroids should observe the client for adverse effects of: (Select all that apply.) Your Answer: Hyponatremia. Correct Answers: Increase in intraocular pressure. Pathologic fractures. Rationale: Corticosteroids accelerate osteoporosis and increase the risk for pathologic fractures. Corticosteroids result in sodium retention (not hyponatremia), which causes fluid retention, which could result in increased intraocular pressure. Weight gain, not weight loss, is a side effect of corticosteroid therapy. Integrated Process: Nursing Process; Assessment Cognitive Level: Evaluation NCLEX-RN Test Plan: Physiological Integrity;

Pharmacological and Parenteral ________________________________________ 8. Which of the following medications should the nurse anticipate the physician would prescribe for "rescue" from sudden-onset wheezing, tightness in the chest, or shortness of breath due to an asthma diagnosis? Your Answer: (blank) ________________________________________ 9. Which of the following interventions would be a priority in the plan of care for the person with pulmonary emphysema? (Select all that apply.) Your Answer: Maintaining hydration status Correct Answer: Low-flow oxygen via face mask Rationale: Patients with pulmonary emphysema have a low pO2 that keeps them alive (their hypoxic drive). High levels of oxygen would diminish their response to breathe. Persons with pulmonary emphysema have narrowed airways. Aminophylline is a bronchodilator that works to open narrowed airways and improve airflow and gas exchange. Patients with pulmonary emphysema often have respiratory infection, and are treated with broad-spectrum antibiotics, like ampicillin. These patients often are dehydrated, and need secretions liquefied for easier expectoration. Limiting fluids would help decrease airway clearance and therefore diminish gas exchange. Integrated Process: Nursing Process; Analysis

Cognitive Level: Analysis NCLEX-RN Test Plan: Physiological Integrity; Physiological Adaptation ________________________________________ 10. A patient has been instructed in pursed-lip breathing. The patient asks the nurse the purpose behind this breathing pattern. The nurse's best response to this question would be: Your Answer: "Pursed-lip breathing prevents airway collapse and enhances effective breathing." Rationale: Pursed-lip breathing during exhalation is a useful technique for preventing early airway collapse. Integrated Process: Teaching/Learning Cognitive Level: Analysis NCLEX-RN Test Plan: Psychosocial Integrity ________________________________________ 11. During an initial interview, the charge nurse informs a client that even though he is the youngest client in the nursing home, he will be required to participate in activities with older patients. He tells the nurse that he can handle the interaction, but will need the nurse's help during the initial adjustment. The responses from the patient indicate which of the following? Your Answer: A stressful situation in which challenge and gain are anticipated Rationale: Selye believes that it is not just the "bad" things in life that cause stress, but the "good"

things as well. An effective coping strategy is drawing strength from adversity. Integrated Process: Caring Cognitive Level: Synthesis NCLEX-RN Test Plan ________________________________________ 12. Which of the following measures is the most important action in preventing the transmission of pulmonary tuberculosis? Your Answer: Adequate ventilation of rooms Rationale: Pulmonary tuberculosis is an airborne disease that is spread from person to person by droplets when an infected person coughs, sneezes, speaks, sings, or laughs. Responses 1, 2, and 3 are not adequate interventions for the prevention of tuberculosis. Having rooms with adequate ventilation of non-recirculated air is the best plan. Integrated Process: Nursing Process; Planning Cognitive Level: Analysis NCLEX-RN Test Plan: Health Promotion and Maintenance ________________________________________ 13. Clients identified with pulmonary tuberculosis due to M. tuberculosis usually are placed on Isoniazid (INH) to prevent active disease. Client teaching prior to discharge from a clinic setting would place priority on which of the following post-discharge instructions? Your Answer: Follow-up care will include

reading a PPD test within 72 hours. Correct Answer: Liver function tests should be monitored routinely. Rationale: Isoniazid causes hepatotoxicity; therefore, persons on Isoniazid should have liver function tests routinely monitored. After diagnosis of TB, a PPD always will be positive. Headache and weight gain are not priorities. Integrated Process: Teaching Learning Cognitive Level: Analysis NCLEX-RN Test Plan: Physiological Integrity; Pharmacological Therapies ________________________________________ 14. Patients with suspicious masses in the lung as seen on chest x-ray could have which of the following diagnostic studies to provide information on soft tissue masses? Your Answer: Pulmonary function tests (PFTs) Correct Answer: CAT scan and magnetic resonance imaging (MRI) Rationale: The chest x-ray usually is the first examination that the physician will order. Suspicious masses seen on the chest x-ray could signal the need for CAT scan and an MRI to provide additional information about soft tissue masses. The gastrograffin swallow test would be used to diagnose an esophageal rupture, while a wedge resection would be treatment for a lung mass. Integrated Process: Nursing Process; Analysis Cognitive Level: Analysis

NCLEX-RN Test Plan: Physiological Integrity; Reduction of Risk Potential ________________________________________ 15. A patient is being treated for community-acquired pneumonia. Which of the following drugs should the nurse question, related to this treatment? Your Answer: Amoxicillin (Amoxil) Correct Answer: Chlorzoxazone (Paraflex) Rationale: Chlorzoxazone is a skeletal muscle relaxant. Erythromycin, azithromycin, and amoxicillin are oral antibiotics recommended to treat older persons with community-acquired pneumonia. Integrated Process: Nursing Process; Evaluation Cognitive Level: Analysis NCLEX-RN Test Plan: Physiological Integrity; Pharmacological and Parenteral Therapies One of the challenges in meeting the nutritional needs of the elderly is that the elderly: Your Answer: Have a decreased need for almost all nutrients. Correct Answer: Have decreased caloric needs but constant or increased needs for vitamins and minerals. Rationale: A decreasing metabolic rate (or resting energy expenditure) means fewer calories are required, but the DRI for most nutrients remains unchanged or might be increased in the elderly,

necessitating careful dietary planning to meet those needs. Loss of interest in food might mean physiologic problems exist. Dental problems for the elderly are not nearly as common today as they were a generation ago. Nursing Process: Planning Cognitive Level: Analysis NCLEX-RN Test Plan: Physiological Integrity ________________________________________ 2. When an elderly person is hospitalized with pneumonia, what would be the best action by the nurse to encourage eating? Your Answer: Secure an order for a daily multivitamin to stimulate appetite. Correct Answer: Provide small, more frequent meals. Rationale: An elderly person is more likely to eat if portions are small, more frequent, and nutrient-dense, as opposed to a full meal tray three times a day. A vitamin will not stimulate appetite. Snacks are not necessary if small, frequent meals are provided. Hydration is always important, but fluids do not provide the nutrients needed for healing. Nursing Process: Implementation Cognitive Level: Application NCLEX-RN Test Plan: Health Promotion and Maintenance ________________________________________ 3. What preventive measures might the nurse teach

the elderly to prevent constipation? Your Answer: Drink coffee or tea only in the morning. Correct Answer: Drink a glass of water several times a day. Rationale: Adequate fluid intake, along with adequate fiber, are the best dietary measures to prevent constipation. Daily stool softeners can lead to dependency on them for bowel movements. Chewing is important, but will not prevent or control constipation. Coffee or tea taken only once per day will not supply enough fluid to prevent constipation. Nursing Process: Planning Cognitive Level: Application NCLEX-RN Test Plan: Physiological Integrity ________________________________________ 4. What type of diet will the nurse recommend to most healthy elderly clients? Your Answer: A diet low in fat and protein Correct Answer: A diet high in complex carbohydrates and fiber Rationale: Complex carbohydrates and fiber provide some protein in addition to some necessary vitamins and minerals. They aid digestion, and have a lower glycemic load. For an active, healthy elderly person, 1,200 calories per day will not be adequate to prevent weight loss. Inadequate protein does not allow for tissue maintenance and repair. Highcarbohydrate diets do not provide adequate balance to caloric distribution.

Nursing Process: Planning Cognitive Level: Application NCLEX-RN Test Plan: Health Promotion and Maintenance ________________________________________ 5. An elderly client has been admitted to a nursing home, and the nurse completes an assessment. Which finding might lead the nurse to suspect a nutritional alteration? Your Answer: Moist conjunctiva Correct Answer: Ridged nails Rationale: Ridged, spoon-shaped nails are signs of long-term nutritional deficiencies. Shiny hair and moist conjunctiva are signs of nutritional balance. A pale tongue, in the absence of other pathology, does not signify nutritional inadequacy. Nursing Process: Assessment Cognitive Level: Analysis NCLEX-RN Test Plan: Physiological Integrity ________________________________________ 6. The elderly client who wants to take an herbal supplement for arthritis symptoms should be advised to: Your Answer: Verify their effectiveness with friends or family members who have taken them. Correct Answer: Consult their healthcare provider about possible interactions with current medications. Rationale: Herbal supplements might be of

unknown quality, and also might interact with medications. Serious side effects are more likely when used with some prescription medications. Family and friends are not the best source of health information. Nursing Process: Evaluation Cognitive Level: Analysis NCLEX-RN Test Plan: Health Promotion and Maintenance ________________________________________ 7. The nurse who is preparing to insert a nasogastric tube for enteral feedings recognizes that this intervention is used for elderly clients who: Your Answer: Cannot adequately metabolize nutrients. Correct Answer: Are unable to ingest food. Rationale: Enteral support is used for clients who are unable to ingest adequate amounts of food for tissue maintenance and repair. Digestion occurs in the GI tract, and metabolism occurs at the cellular level. The gag reflex is not related to nutritional support unless it is present as a result of a medical condition; a CVA, for example. Nursing Process: Implementation Cognitive Level: Analysis NCLEX-RN Test Plan: Physiological Integrity ________________________________________ 8. When working with elderly clients who require an increased consumption of complete protein, the nurse recommends:

Your Answer: Correct Answer:

Iron-fortified cereal. 2% milk.

Rationale: After eggs and meats, milk is the best source of complete protein. Legumes, cereal, and bread have some protein, but it is incomplete protein. Nursing Process: Planning Cognitive Level: Knowledge NCLEX-RN Test Plan: Health Promotion and Maintenance ________________________________________ 9. An appropriate dietary intervention for an elderly client with dysphagia is: Your Answer: Thicken thin liquids. Rationale: Dysphagia means difficulty with swallowing. Liquids that are thickened are easier to swallow and less likely to cause the client to choke or gag. The frequency or temperature of foods does not affect the ability to swallow. Nursing Process: Implementation Cognitive Level: Application NCLEX-RN Test Plan: Physiological Integrity ________________________________________ 10. A primary goal of dietary intervention for the elderly is to: Your Answer: Improve digestion and metabolism. Correct Answer: Maintain quality of life.

Rationale: Dietary assessment and early intervention can improve overall quality of life and possibly prevent disease or illness. Most acute conditions are not treated primarily by dietary intervention. Digestion occurs in the GI tract, and metabolism occurs at the cellular level. Nursing Process: Implementation Cognitive Level: Analysis NCLEX-RN Test Plan: Health Promotion and Maintenance ________________________________________ 11. Which nutrient is most likely to be deficient in elderly residents of nursing homes? Your Answer: Protein Correct Answer: Vitamin D Rationale: The elderly have greater need for vitamin D due to decreased absorption, decreased dietary intake, and lack of exposure to sunlight. Calcium, protein, and vitamin K are more readily obtained through dietary means. Nursing Process: Evaluation Cognitive Level: Comprehension NCLEX-RN Test Plan: Health Promotion and Maintenance ________________________________________ 12. The loss of lean muscle mass that occurs with aging can be diminished or reversed by: Your Answer: Small doses of anabolic hormones. Correct Answer: Strength-training exercises.

needed. Rationale: Resistance exercises, such as lifting weights, have been shown to increase muscle mass in the elderly. Anaerobic exercises, highprotein diets, and anabolic steroids are not effective in reducing loss of muscle mass or increasing muscle mass. Nursing Process: Evaluation Cognitive Level: Application NCLEX-RN Test Plan: Physiological Integrity ________________________________________ 13. The absorption of iron in the elderly can be diminished in the presence of: Your Answer: Vitamin C. Correct Answer: Antacids. Rationale: Anything that interferes with the production of hydrochloric acid interferes with the absorption of iron, and should be avoided when trying to promote iron absorption. Diuretics, vitamin C, and milk do not affect iron absorption. Nursing Process: Implementation Cognitive Level: Knowledge NCLEX-RN Test Plan: Physiological Integrity ________________________________________ 14. When serving a meal to elderly residents of a nursing home, the nurse should: Your Answer: Open cartons and cut food into bite-sized portions. Correct Answer: Provide adaptive equipment as Rationale: The use of adaptive equipment allows residents to be as independent as possible at mealtime. It is important to allow clients to do as much as possible for themselves. Nursing Process: Implementation Cognitive Level: Application NCLEX-RN Test Plan: Safe, Effective Care Environment ________________________________________ 15. A major predictor of morbidity and mortality in the elderly is: Your Answer: Polypharmacy. Correct Answer: Unintentional weight loss. When an elderly male client states he has pain, the nurse recognizes that: Your Answer: The client probably has significant pain, because many elderly understate and under-report pain. Rationale: Many elderly fear being labeled as complainers, so they frequently fail to mention having pain, or diminish its significance to them. The elderly are no more prone to exaggerate pain than is any other age group. The elderly do not lose pain receptors, and the most frequent reason for healthcare visits is to manage chronic health conditions. Nursing Process: Assessment Cognitive Level: Application

NCLEX-RN Test Plan: Physiological Integrity ________________________________________ 2. When the family of an elderly resident at the nursing home wants to file a complaint about the improper use of restraints, the nurse should suggest that they: Your Answer: Contact the attorney retained by the nursing home administrator. Correct Answer: Call the ombudsman assigned to the nursing home. Rationale: The improper use of restraints is a valid reason for contacting the nursing home ombudsman, whose name and phone number should be readily available to residents and families. The other options do not identify the appropriate individual to handle the familys request to file a complaint. Nursing Process: Evaluation Cognitive Level: Knowledge NCLEX-RN Test Plan: Safe, Effective Care Environment ________________________________________ 3. The nurse who understands the Patient SelfDetermination Act of 1990 understands that: Your Answer: Medicare coverage is extended for those elderly who suffer from catastrophic conditions. Correct Answer: Elderly clients who are competent have the right to make informed decisions regarding healthcare.

Rationale: This federal law governs anyones ability to give informed consent after being fully informed about the options, benefits, or risks of any given treatment. The other options might be desirable; they are not part of the Patient SelfDetermination Act of 1990. Nursing Process: Planning Cognitive Level: Knowledge NCLEX-RN Test Plan: Safe, Effective Care Environment ________________________________________ 4. Which of the following will the nurse want to include when planning a functional assessment of an elderly client? Your Answer: A social worker Rationale: A functional assessment is conducted by an interdisciplinary team, and may be performed while a client is an inpatient at a hospital, but also may be conducted in an office setting. All of the other things might be nice to have when the assessment is conducted, but the nurse does not need to have them. Nursing Process: Planning Cognitive Level: Application NCLEX-RN Test Plan: Health Promotion and Maintenance ________________________________________ 5. When a nursing home uses the Minimum Data Set (MDS), the nurse is assured that:

Your Answer: Regular dental care is being provided. Correct Answer: The nursing home is meeting federal regulations. Rationale: The MDS provision of the Omnibus Budget Reconciliation Act of 1987 requires assessment of all residents using the criteria of the MDS. The MDS does not address the other options. Nursing Process: Evaluation Cognitive Level: Analysis NCLEX-RN Test Plan: Safe, Effective Care Environment ________________________________________ 6. The primary purpose for documenting an adverse event, such as a fall, is to: Your Answer: Record unusual occurrences. Correct Answer: Improve the quality of care. Rationale: Adverse events are always opportunities for quality improvement, and documentation should lead to system analysis and improvement. Careful records need to be kept to document follow-up with respect to staff development and client outcomes; however, the ultimate goal is to improve the quality of care. Nursing Process: Evaluation Cognitive Level: Analysis NCLEX-RN Test Plan: Safe, Effective Care Environment ________________________________________ 7.

Which of the following nursing actions will help maintain safety for the elderly? Your Answer: Encourage independence. Correct Answer: Orient to new surroundings. Rationale: Safety is a priority need for the elderly, especially when in new surroundings, such as a hospital or long-term care facility; they can become disoriented when they cannot easily locate familiar places like the bathroom. The nursing actions in the other actions are desirable but are not primarily for client safety. Nursing Process: Implementation Cognitive Level: Application NCLEX-RN Test Plan: Safe, Effective Care Environment ________________________________________ 8. The nurse observes the following actions and recognizes that a breech of confidentiality is evident when a colleague: Your Answer: Uses the phone to confer with a physicians nurse about a clients change of condition. Correct Answer: Throws unofficial lab results into the regular trash. Rationale: Client information may be shared with those who need to know it. When information with a clients name is thrown in the regular trash, it might be found and read by anyone. Nursing Process: Evaluation Cognitive Level: Application

NCLEX-RN Test Plan: Safe, Effective Care Environment ________________________________________ 9. When an elderly person agrees with a physician that a do not resuscitate (DNR) order is appropriate, that person is said to be: Your Answer: In need of a living will. Correct Answer: Competent. Rationale: When a physician explains to a client that additional care is futile, the client who is competent might not agree. Based on the wishes of the competent client, an order may be written. Giving assent suggests that the client cannot give fully informed consent. The competent client has full decisional capacity, and does not need a living will unless he so chooses. Nursing Process: Implementation Cognitive Level: Application NCLEX-RN Test Plan: Safe, Effective Care Environment ________________________________________ 10. When obtaining a social history from an elderly client, the nurse will ask about: Your Answer: Problems with memory, judgment, and thought. Correct Answer: Living arrangements and family dynamics. Rationale: A social history has to do with social support systems and interpersonal

relationships. The other choices presented are dealt with in a functional assessment. Nursing Process: Assessment Cognitive Level: Knowledge NCLEX-RN Test Plan: Psychosocial Integrity Primary prevention of elder mistreatment is a healthcare goal. The nurse recognizes that one of the risk factors often present in elder mistreatment situations is: Your Answer: Caregiver independence. Correct Answer: Elder dependency. Rationale: One of the primary characteristics in elders who have been mistreated is dependency on others for performance of activities of daily living. Dependence can lead to caregiver strain and then mistreatment. Home care is not a risk factor for mistreatment. Caregiver independence is not related to mistreatment of the elderly. Elders who have minimal family support can feel abandoned. Nursing Process: Analysis Cognitive Level: Comprehension NCLEX-RN Test Plan: Psychosocial Integrity ________________________________________ 2. A home care nurse found an elderly client with multiple bruises on both arms and the trunk. After the client denied the spouse was responsible, the rationale the nurse used for reporting the suspicion of abuse was: Your Answer: The client was too afraid to report the spouse.

Correct Answer: Reporting suspected abuse is an ethical responsibility of the nurse. Rationale: The nurse has an ethical and legal responsibility to report suspected abuse to designated authorities for investigation. Because the nurse has this responsibility, legal liability is not an issue, nor are the denials by the client grounds for not reporting the suspicion of abuse. Nursing Process: Analysis Cognitive Level: Application NCLEX-RN Test Plan: Safe, Effective Care Environment ________________________________________ 3. When a nurse determines with a high degree of confidence that an elderly homebound client has been mistreated, an appropriate intervention would be: Your Answer: Consulting with the physician about admission to the hospital for a thorough assessment of the client. Rationale: It is appropriate to admit a client to the hospital for a thorough assessment in a safe environment, and it serves as an immediate way to remove the client from the situation. Social Services will conduct an investigation, but the nurse has a responsibility to formulate a plan of care for the client. The caregiver might not be responsible for the abuse, so it would not be appropriate to suggest a new caregiver. Police surveillance is not done for suspected mistreatment.

Nursing Process: Implementation Cognitive Level: Analysis NCLEX-RN Test Plan: Safe, Effective Care Environment ________________________________________ 4. Which of the following situations could be the source of an ethical dilemma for the nurse with respect to elder mistreatment? Your Answer: The family sues the nurse for reporting abuse. Correct Answer: The elderly person decides to return to the abusive setting. Rationale: The nurse recognizes an ethical dilemma is posed when a competent adult chooses to return to the setting in which abuse occurred, because there is a high likelihood of abuse being repeated. The elderly person who is not competent will not be allowed to make decisions about the living situation. Injury is not a source of an ethical dilemma, nor is legal action taken against a nurse. Nursing Process: Assessment Cognitive Level: Application NCLEX-RN Test Plan: Safe, Effective Care Environment ________________________________________ 5. An elderly person reports all of the following experiences to the nurse. Which of the reported experiences is the most likely indicator of exploitation of the elderly person? Your Answer: The client is being threatened

with admission to a nursing home. Correct Answer: There isnt any money available to make a weekly donation at church. Rationale: When the elderly are exploited, others take advantage of them, almost always with respect to money matters. The exploiters might not be family members. Failure of family to visit and forced attendance of family events are not exploitation. Threats can be attempts at coercion but are not exploitation. Nursing Process: Analysis Cognitive Level: Analysis NCLEX-RN Test Plan: Psychosocial Integrity ________________________________________ 6. The nurse suspects that a home health aide who comes into the home every day to provide basic hygiene care is abusing an elderly client. What is the best way to gather more information about the situation to confirm suspicions of abuse? Your Answer: Interview the client in private. Rationale: The best method of getting information is to interview the client in a private setting. Depending on the responses of the client, the nurse can develop a plan for how to proceed. The nurse then can consult the family or involve a social services agency. Nursing Process: Assessment Cognitive Level: Application NCLEX-RN Test Plan: Health Promotion and Maintenance

________________________________________ 7. What data would help support a nursing diagnosis of elder mistreatment in the domestic setting? Your Answer: Client has a flat affect. Correct Answer: Caregiver will bathe the client only once a week. Rationale: For the elderly, some hygiene measures might not be needed or desirable in the same frequency as for younger adults. However, bathing only once weekly is not likely to be sufficient. Unhappiness with living arrangements and a flat affect are not evidence of mistreatment. Recognition of challenges with care giving does not mean mistreatment has occurred. Nursing Process: Assessment Cognitive Level: Analysis NCLEX-RN Test Plan: Safe, Effective Care Environment ________________________________________ 8. Mistreatment that occurs in nursing homes and is directed toward the elderly often is due to: Your Answer: Staffs lack of interest in the elderly as individuals. Correct Answer: Low wages and heavy workloads of the nursing assistants. Rationale: Individuals who have few skills and inadequate education in the complex care needs of the elderly, along with accompanying low wages and heavy workloads, might mistreat the elderly out

of simple frustration. Nursing Process: Evaluation Cognitive Level: Analysis NCLEX-RN Test Plan: Safe, Effective Care Environment ________________________________________ 9. A nurse who plans to do research on caregiver strain in spouses of homebound elderly with cognitive impairment due to dementia is most likely to choose which theory of elder mistreatment to guide the study? Your Answer: Situational theory Rationale: Situational theory examines the burdens of caregiving and the ability of the caregiver to meet the needs and demands of the elderly individual. Mistreatment can result when the situation becomes overwhelming for the caregiver. The other theories do not relate to caregiver strain. Nursing Process: Planning Cognitive Level: Application NCLEX-RN Test Plan: Health Promotion and Maintenance ________________________________________ 10. Which of the following observations is indicative of physical mistreatment? Your Answer: Downcast eyes Correct Answer: Dislocated shoulder Rationale: This is the only listed physical sign. A dislocated shoulder can occur with a violent jerk

on the arm. Nursing Process: Assessment Cognitive Level: Analysis NCLEX-RN Test Plan: Psychosocial Integrity An elderly client whose middle-age daughter recently died of breast cancer now complains of mild abdominal pain, five-pound weight loss, insomnia, and fatigue. When no physiological cause can be found, the nurse suspects these are symptoms of: Your Answer: Hypochondria. Correct Answer: Normal grieving. Rationale: Physiological symptoms are a normal part of grieving, particularly in the early phases. In the absence of pathology, these are signs of normal grieving. Denial, spiritual distress, and hypochondria usually do not manifest themselves with physical symptoms. Nursing Process: Evaluation Cognitive Level: Analysis NCLEX-RN Test Plan: Psychosocial Integrity ________________________________________ 2. A nurse who works effectively with elderly clients who are dying and their families recognizes that: Your Answer: At least some pain accompanies most deaths. Correct Answer: The nurse must be comfortable with her own concerns and feelings about death. Rationale: Nurses and other healthcare providers who are comfortable with their own

understanding of death can be effective when working with the dying, rather than bringing their conflicted feelings to a stressful situation. Unless the nurse has used some critical thinking to look at and investigate her feelings and understandings, they might conflict with those of the client. Many people are indeed afraid of dying, especially if pain might occur. However, pain is not inevitably associated with death. Hospice might not be appropriate or possible for many individuals. Nursing Process: Analysis Cognitive Level: Comprehension NCLEX-RN Test Plan: Psychosocial Integrity ________________________________________ 3. It is important for a nurse to understand the grieving process because: Your Answer: It assists the nurse to understand the dynamics of grieving. Rationale: Grief is unique to each individual and family and culture. By understanding the grieving process, the nurse will recognize the social, cultural, and physiological dynamics present during grieving. There is no optimal order in which individuals must move through the grieving process. Nursing Process: Evaluation Cognitive Level: Comprehension NCLEX-RN Test Plan: Psychosocial Integrity ________________________________________ 4. The goal of nursing interventions for a bereaved elderly person is to:

Your Answer: Assist the bereaved individual to achieve a healthy adjustment to the loss. Rationale: The goal of grieving is to adjust to the loss in a time and manner that is culturally acceptable to the individual who experienced the loss. There is no timetable that must be met, no defined way to achieve the goal. An intervention to progress toward the goal is verbalization about the loved one. Teaching is a nursing goal and is not client-centered. Nursing Process: Planning Cognitive Level: Comprehension NCLEX-RN Test Plan: Psychosocial Integrity ________________________________________ 5. An elderly person in end-stage renal disease is admitted to a nursing home for palliative care. Nursing interventions will be: Your Answer: Ambulation as desired. Correct Answer: Pain relief. Rationale: The purpose of palliative care is pain relief when no additional treatment is desired or available. Comfort measures direct the primary nursing intervention, and can include medications or complementary therapies, such as massage. Individuals receiving palliative care are not candidates for CPR. Urine output is not expected in end-stage renal disease. Nursing Process: Planning Cognitive Level: Application NCLEX-RN Test Plan: Physiological Integrity

________________________________________ 6. A nurse is uncomfortable discussing spiritual concerns with a dying client. The most helpful action for the client would be for the nurse to plan to: Your Answer: Request a member of the pastoral care staff visit the client. Rationale: Pastoral care is available at most sites, and if a nurse is uncomfortable with spiritual issues, the pastoral care services might be better able to meet the spiritual needs of the client. A nurse would plan to get education, rather than counseling, on how to meet the needs of dying clients. Discomfort with a situation is not a reason for asking to be removed from caring for that client. Nursing Process: Planning Cognitive Level: Application NCLEX-RN Test Plan: Psychosocial Integrity ________________________________________ 7. When opioids are prescribed for pain at the end of life, the nurse should understand that: Your Answer: Opioids most likely will be a PRN order. Correct Answer: Side effects still must be treated. Rationale: Opioids are the gold standard of pain relief; however, side effects like constipation are common, and must be treated to provide comfort to the client. A client can be on opioids for a long time before death; they are most effective when given on an around-the-clock schedule. A

client may be on other medications concurrently. Nursing Process: Evaluation Cognitive Level: Comprehension NCLEX-RN Test Plan: Physiological Integrity ________________________________________ 8. A priority nursing intervention for an elderly person who is dying and experiencing anxiety is to: Your Answer: Allow the client time alone to conduct a life review. Correct Answer: Assist the individual to identify fears. Rationale: People who are dying have many fears, including pain, being left alone, or being forgotten. The nurse who can help the client identify those fears then can work with the client to explain what can be done to alleviate the concerns. Some time alone might be appropriate, but many times, it increases anxiety. Contacting family might be appropriate, but not until after the nurse has determined with the client that it will be helpful. Merely explaining that anxiety is common, without further intervention, is not the best action the nurse can take. Nursing Process: Implementation Cognitive Level: Application NCLEX-RN Test Plan: Psychosocial Integrity ________________________________________ 9. Which of the following statements, if made by a dying client, would indicate that spiritual needs most likely are being met? The individual states

that: Your Answer: There have been many positive things about my life, and I have hope. Rationale: When an individuals spiritual needs have been met, the person can be expected to express feelings of satisfaction with ones life and develop a new understanding of hope. Many people who do not subscribe to particular spiritual beliefs might not fear pain, and many have an appreciation for family. Some religious groups do not believe in an afterlife. Nursing Process: Evaluation Cognitive Level: Analysis NCLEX-RN Test Plan: Psychosocial Integrity ________________________________________ 10. When an elderly client expresses a wish to forgo additional treatment for cancer and to die, a priority action of the nurse would be: Your Answer: Explore the clients understanding of the consequences of such a decision. Rationale: When respecting a clients autonomy, the nurse recognizes that the client has a right to make such a decision. It is important to make sure the client understands what such a decision means about the outcome of the disease process. At some point, the physician and family should be notified, but only after discussion with the client affirms the decision. The role of the nurse is to support a client, not suggest what the client should

do. Nursing Process: Implementation Cognitive Level: Application NCLEX-RN Test Plan: Psychosocial Integrity

When administering medication to an elderly hospitalized client, the nurse should be aware that there is an increased possibility of drug toxicity due to which of the following age-related changes? Your Answer: Decreased filtration and excretion of drug metabolites through the renal system Rationale: Glomerular filtration and creatinine clearance begin to decline in middle age, and can be significantly lower in most elderly, posing a risk of greater drug toxicity. There is no increase in protein-binding sites or circulating body fluids in the elderly. Production of enzymes also is not decreased with aging. Nursing Process: Planning Cognitive Level: Application NCLEX-RN Test Plan: Physiological Integrity ________________________________________ 2. Which of the following indicates that an elderly client has been affected by polypharmacy? Your Answer: Medications are used to counteract side effects of currently prescribed medications.

Rationale: Polypharmacy means that multiple medications have been prescribed. When medications are prescribed to counteract side effects of other medications, it is an indication that a total review of medications should occur to determine the optimum medications and dosages with the fewest side effects. Lack of improvement in a condition might mean another evaluation is warranted. Healthcare providers need to determine the possible benefits and risks in prescribing or not prescribing any given medication. Nursing Process: Evaluation Cognitive Level: Analysis NCLEX-RN Test Plan: Health Promotion and Maintenance ________________________________________ 3. In assessing use of over-the-counter (OTC) medications, which of the following questions will the nurse want to ask the elderly client? Your Answer: Who pays for the medication? Correct Answer: How does this medication help you? Rationale: It is important to assess how the medication helps the client or how the client feels it is helpful. Payment or brand selections are up to the client, and might be related to other considerations. Other treatment strategies do not relate to OTC medications. Nursing Process: Assessment Cognitive Level: Application NCLEX-RN Test Plan: Health Promotion and

Maintenance ________________________________________ 4. What is frequently a contributing factor to the elderly using multiple medications? Your Answer: Increased challenges in compliance with medication regimens Correct Answer: Healthcare providers failure to coordinate medication regimens Rationale: The elderly client might be under the care of multiple healthcare providers who are unaware of the medications the other providers have prescribed. The elderly should be encouraged to maintain a list of medications, dosages, frequency, and prescriber information to share with all healthcare providers. The greater the number of medications the client is taking, the higher the likelihood of side effects and decreased compliance with medication regimens. Nursing Process: Evaluation Cognitive Level: Analysis NCLEX-RN Test Plan: Health Promotion and Maintenance ________________________________________ 5. Anticholinergic effects of antipsychotic medications in the elderly are associated with: Your Answer: Cognitive impairment. Correct Answer: Difficulty with voiding. Rationale: Anticholinergic effects commonly are found in prescribed medications, and frequent

side effects (urinary retention, dry mouth, blurred vision, decreased peristalsis, etc.) occur in the elderly, so the nurse needs to assess for those side effects with elderly clients. Nursing Process: Assessment Cognitive Level: Knowledge NCLEX-RN Test Plan: Physiological Integrity ________________________________________ 6. Gastritis in the elderly frequently has been associated with: Your Answer: Beta blockers. Correct Answer: Low doses of aspirin. Rationale: Aspirin is highly irritating to the stomach, and can lead to heartburn or gastritis, even with low-dose or enteric coating, if it is not taken properly. Other types of medications listed are not frequently associated with gastritis. Nursing Process: Implementation Cognitive Level: Application NCLEX-RN Test Plan: Physiological Integrity ________________________________________ 7. General guidelines regarding prescribing medications for the elderly clients are: Your Answer: Start low and go slow. Rationale: This rule of thumb is followed to prevent toxic effects and adverse drug reactions. The usual practice is to start with the lowest dose possible and evaluate regularly to see if symptoms subside or the condition improves; the dose is

increased only if necessary. Nursing Process: Planning Cognitive Level: Comprehension NCLEX-RN Test Plan: Health Promotion and Maintenance ________________________________________ 8. The ultimate goal of prescribing antipsychotic medications for the elderly is: Your Answer: To relieve stress on family members who are caregivers. Correct Answer: To improve the quality of life. Rationale: Antipsychotic medications should be prescribed with great caution in the elderly and should be viewed only as short-term therapy to improve quality of life. Nursing Process: Implementation Cognitive Level: Analysis NCLEX-RN Test Plan: Psychosocial Integrity ________________________________________ 9. An elevated level of a highly protein-bound medication might be a result of which pharmacokinetic alteration in the elderly? Your Answer: Increased CPK Correct Answer: Decreased serum albumin Rationale: Decreased serum albumin results in greater amounts of the unbound proportion of protein-bound medications. This can result in toxic levels, because the unbound portion is available to produce effects. The other substances do not affect

protein-bound medications. Nursing Process: Planning Cognitive Level: Knowledge NCLEX-RN Test Plan: Health Promotion and Maintenance ________________________________________ 10. The area in which the nurse may have the greatest effect on medication use in the elderly is: Your Answer: Screening for side effects. Correct Answer: Client education. Rationale: Teaching clients and families about safe practices and information about side effects will ensure medications that are effective are used correctly and have the fewest effects. The nurse may encourage alternative therapies if they do not affect medication compliance or increase the risk of side effects. Monitoring clients is important, but teaching is likely to have the greatest impact on clients. Nursing Process: Implementation Cognitive Level: Analysis NCLEX-RN Test Plan: Health Promotion and Maintenance ________________________________________ 11. When developing a teaching plan for an elderly client newly diagnosed with hypertension, the nurse realizes that the most effective teaching strategy is: Your Answer: Sharing the Web address of the American Heart Association. Correct Answer: Providing oral instructions along

with a written copy to take home. Rationale: Oral instruction allows the nurse to answer questions and to verify client understanding of instructions. Written information is for the client or family to review if they have additional questions or need more information. Clients might seek other sources of information, but it is the responsibility of the nurse to provide information that is understandable to the client. Nursing Process: Implementation Cognitive Level: Application NCLEX-RN Test Plan: Health Promotion and Maintenance ________________________________________ 12. An elderly client asks why he seems to respond differently to some medications than he did in the past. The nurse should keep in mind that: Your Answer: In the elderly, an adverse drug reaction can occur even after the medication has been discontinued. Rationale: Due to the pharmacokinetics and pharmacodynamics in the elderly, it is possible to have adverse reactions during or following discontinuation of medication therapy. Glomerular filtration is decreased in the elderly, so there might be more side effects. Nursing Process: Evaluation Cognitive Level: Analysis NCLEX-RN Test Plan: Health Promotion and Maintenance

________________________________________ 13. If an elderly client tells the nurse about medications from an internet site with NAPB VIPPS (National Association of Boards of Pharmacy, Verified Internet Pharmacy Practice Site), the nurse can be fairly certain that: Your Answer: The client has financial need. Correct Answer: The site meets state and federal requirements. Rationale: Increasing use of the internet by the elderly and others to purchase medications has led to federal and state regulations that assure safety for the consumer. Medications purchased from the internet are not necessarily from foreign countries, and do not necessarily mean the client is experiencing financial distress. Nursing Process: Evaluation Cognitive Level: Analysis NCLEX-RN Test Plan: Health Promotion and Maintenance You are the nurse assigned to perform an eye assessment on an 80-year-old client. Which of the following findings during the assessment is considered normal? Your Answer: Absence of the red reflex Correct Answer: Equal pupillary constriction in response to light Rationale: Pupils are small in old age, and the pupillary light reflex can be slowed, but pupillary

constriction should be symmetrical. There should be a red reflex, and the eyelids and conjunctivae should not be edematous or inflamed. Integrated Process: Nursing Process; Assessment Cognitive Level: Analysis NCLEX-RN Test Plan: Physiological Integrity; Reduction of Risk ________________________________________ 2. Which of the following physiologic changes would be expected in a patient with presbyopia? Your Answer: Loss of lens elasticity Rationale: The lens loses elasticity or pliability, decreasing its ability to change shape in order to accommodate for near vision. Response 4 is indicative of cataracts, 3 is myopia, and 1 is due to corneal changes. Integrated Process: Nursing Process; Assessment Cognitive Level: Analysis NCLEX-RN Test Plan: Physiological Integrity; Reduction of Risk Potential ________________________________________ 3. A patient has a question about a recent eye exam. Which of the following statements would be an accurate response to inquiry? Your Answer: "The lens is normally transparent." Rationale: The lens is made of layers of epithelial cells that are closely packed and organized so that the lens is clear and transparent.

Hypermetropia is a shortened eyeball, and myopia is an elongated eyeball. Refraction refers to bending the light rays so they are focused at a certain point on the retina. Integrated Process: Teaching Learning Cognitive Level: Application NCLEX-RN Test Plan: Health Promotion and Maintenance ________________________________________ 4. A client comes to the office with a tentative diagnosis of glaucoma, and is to have testing to determine whether the anterior angle of the eye is open or closed. The patient will require education for which of the following examinations? Your Answer: Retinography Correct Answer: Slit-lamp inspection Rationale: The slit-lamp inspection of the iris is done to assess whether the anterior angle of the eye is open or closed. This is part of a complete dilated examination to inspect the optic nerve and retina. Tonometry measures only the pressure within the eye. Response 2 would be used to take multiple x-rays of the eye tissue, and retinography records the electrical activity of the eye. Integrated Process: Teaching Learning Cognitive Level: Application NCLEX-RN Test Plan: Health Promotion and Maintenance ________________________________________ 5. A patient with glaucoma has medication prescribed

to decrease intraocular pressure. Which of the following medication should be questioned by the nurse? Your Answer: Artificial tears Rationale: Timoptic, levobunolol, and pilocarpine are drugs usually prescribed for patients with glaucoma and increased intraocular pressure (IOP). Artificial tears are usually prescribed for persons with "dry eyes." Integrated Process: Nursing Process; Evaluation Cognitive Level: Evaluation NCLEX-RN Test Plan: Physiological Integrity; Pharmacological and Parenteral Therapies ________________________________________ 6. A client with history of glaucoma was diagnosed by the community nurse as experiencing Visual Sensory/Perceptual Alterations R/T increased intraocular pressure. The plan of care should focus on: Your Answer: Encouraging compliance with drug therapy to prevent loss of vision. Rationale: Management of glaucoma involves lowering the intraocular pressure to cease drainage to the optic nerve and further vision loss. Therapy involves medications to decrease intraocular pressure. It is critical that the patient understand the importance of compliance with the medication regimen. Glaucoma needs to be controlled to prevent damage that cannot be reversed. Pain is considered an emergency, so if it was identified, the

patient would need immediate intervention. Loss of peripheral vision is a concern, but the priority is to encourage the patient to be compliant with medications to prevent further damage to the vision. Integrated Process: Nursing Process; Implementation Cognitive Level: Synthesis NCLEX-RN Test Plan: Health Promotion and Maintenance ________________________________________ 7. A 76-year-old client tells the nurse that she notices that she is having trouble hearing, especially in large groups. She continues, saying she cannot always tell where the sound is coming from, and the words often sound "mixed up." The nurse should suspect which of the following as the cause for this change? Your Answer: Nerve degeneration in the inner ear Rationale: Hearing loss is common in older adults. Changes in the inner ear involve atrophy of the organ of corti and cochlear, and degeneration of the stria vascularis. This is a gradual hearing loss that affects the auditory nerve, which makes words sound garbled, and the ability to localize sound is also impaired. The result is that communication dysfunction is accentuated when background noise is present. Responses 1, 2, and 4 would not affect the sensorineural component of hearing. Integrated Process: Nursing Process; Analysis

Cognitive Level: Analysis NCLEX-RN Test Plan: Physiological Integrity; Adaptation ________________________________________ 8. You are the nurse assigned to care for an elderly client. While assessing the ears of the patient, you observed dry, hard cerumen developing in the ear canal. Which of the following actions should you take? Your Answer: Irrigate the ear canal with a mixture of hydrogen peroxide and normal saline. Correct Answer: Document finding and report your concern to the physician. Rationale: Cerumen or earwax produced by the ceruminous glands is a normal finding in the ear canal. However, in the older adult, cerumen tends to be dryer and harder, and accumulates in the ear canal due to decreased activity of the apocrine glands. Hearing will become impaired if the cerumen accumulates to impact the canal. Irrigation should be tried after other attempts fail. Therefore, documentation and reporting to the primary care provider will help avoid unnecessary alteration in hearing. Integrated Process: Nursing Process; Analysis Cognitive Level: Analysis NCLEX-RN Test Plan: Physiological Integrity; Basic Care and Comfort ________________________________________ 9. During a hearing assessment, the nurse finds that

sound lateralizes to the client's left ear with the Weber and Rinne tests. What should the nurse conclude from this finding? The patient has: Your Answer: A conductive hearing loss in the right ear. Correct Answer: Either a sensorineural or conductive hearing loss. Rationale: When caring for the older person, a thorough history and physical of the person's hearing is important to help determine the cause of hearing loss, which can be conductive, due to external aspect of the ear, or sensorineural, from an inner ear problem. Both tests determine the degree of hearing loss, if any. Integrated Process: Nursing Process; Assessment Cognitive Level: Analysis NCLEX-RN Test Plan: Physiological Integrity: Reduction of Risk Potential ________________________________________ 10. Which of the following is true of presbycusis? (Select all that apply.) Your Answer: Dysfunction with the cochlea and auditory nerve Correct Answers: Dysfunction with the cochlea and auditory nerve Mnires disease and tumors Infections of the inner ear

Rationale: Presbycusis is hearing loss associated with aging, and affects approximately 75% of people over the age 60. Loss of hair cells in the cochlea (sensory loss) and degradation of neurons (neural loss) result in this form of hearing loss. Presbycusis occurs gradually, and usually is bilateral. Answer 3 describes a conductive hearing loss. Integrated Process: Nursing Process; Analysis Cognitive Level: Application NCLEX-RN Test Plan: Physiological Integrity; Reduction of Risk Potential ________________________________________ 11. An 84-year-old client is admitted to the hospital for treatment of diabetes mellitus. On initial interview, the patient reports having difficulty hearing, and informs the nurse that she has too much ear wax. Because of the patient's age, which of the following questions is most important for the nurse to ask the client in relation to her hearing problem? Your Answer: "What type of diabetes mellitus do you have?" Correct Answer: "Have you ever experienced impacted cerumen?" Rationale: Because the hairs in the ear become coarser during the aging process, retention of wax is more of a cause of hearing loss in older clients. Cerumen, if dry and hard, can impact the ear canal and interfere with hearing. Integrated Process: Nursing Process; Assessment

Cognitive Level: Analysis NCLEX-RN Test Plan: Health Promotion and Maintenance ________________________________________ 12. Which of the following drugs are not associated with impairment? Select all that apply. Your Answer: Gentamicin (Garamycin) Correct Answers: Gentamicin (Garamycin) Cis-platinum (Platinol) Furosemide (Lasix) Rationale: Digoxin (Lanoxin) causes vision changes, but does not affect hearing. Aminoglycosides such as gentamicin, antineoplastics such as cis-platinum, diuretics such as furosemide, antispasmodics such as baclofen, and propranolol cause changes in hearing ability. Integrated Process: Nursing Process; Analysis Cognitive Level: Analysis NCLEX-RN Test Plan: Physiological Integrity; Pharmacological and Parental Therapies ________________________________________ 13. A patient has a normal sensory change that results in diminished sense of taste. How would this be documented? Your Answer: Hypogeusia Rationale: A diminished sense of taste is referred to as hypogeusia. Odynophagia refers to

pain upon swallowing. Xerostomia refers to dry mouth that occurs with salivary gland dysfunction. Arthralgia refers to pain in a joint. Integrated Process: Communication and Documentation Cognitive Level: Analysis NCLEX-RN Test Plan: Safe, Effective Care Environment ________________________________________ 14. A client has noticed a decrease in taste sensation. Which of the following cranial nerves are most likely involved? Your Answer: CN IX and CN VII Rationale: Cranial nerves VII and IX supply sensation to the tongue. Integrated Process: Nursing Process; Analysis Cognitive Level: Analysis NCLEX-RN Test Plan: Physiological Integrity; Reduction of Risk Potential ________________________________________ 15. A 78-year-old client is admitted to the Emergency Department (ED) via emergency medical service (EMS) with complaints of severe diarrhea with resultant weakness and signs of dehydration. Discussion with the significant other reveals that the patient continually eats spoiled foods. Which of the following might be most directly related to this patient's behavior? Your Answer: Damage to cranial nerve III Correct Answer: Damage to cranial nerve I

Rationale: Although hyposmia (impaired sense of smell) might be due to age-related changes, it might also be the result of olfactory nerve damage (cranial nerve I), since this nerve is the sole innervation for smell. Cranial nerve II is responsible for constriction of the pupils, and IV the superior oblique muscle. Cranial II would affect vision, but an intact CN I would allow the individual to identify the odor of spoiled food accurately. Integrated Process: Nursing Process; Analysis Cognitive Level: Analysis NCLEX-RN Test Plan: Physiological Integrity; Physiological Adaptation ________________________________________ 16. In evaluating the functioning of a client's anterior spinothalamic tract, the nurse is testing for: Your Answer: Sense of position. Correct Answer: Light touch sensation. Rationale: It is the spinothalamic tract that transmits light (crude) touch sensation. Integrated Process: Evaluation Cognitive Level: Knowledge NCLEX-RN Test Plan: Health Promotion and Maintenance The nurse is teaching a group of elderly women about skin care. Which of the following would be accurate to include in the content? Your Answer: The dermis is completely replaced every four weeks.

Correct Answer: The dermis contains nerves and sensory receptors. Rationale: The dermis, the second layer of the skin, is made up of connective tissue, collagen, and has resilient elastic tissue that allows the skin to stretch. It is the thickest skin layer and also contains fibroblasts, mast cells, and lymphocytes. Integrated Process: Nursing Process; Planning Cognitive Level: Knowledge NCLEX-RN Test Plan: Health Maintenance and Promotion ________________________________________ 2. During the aging process, the hair can look gray or white and begin to feel thin and fine. This is due to decreased number of functioning: Your Answer: Lymphocytes. Correct Answer: Melanocytes. Rationale: In the aging hair matrix, the number of functioning melanocytes decreases, so the hair looks gray or white and feels thin and fine. The phagocytes, lymphocytes, and eosinophils are cells important to the inflammatory response and the immune system. Integrated Process: Nursing Process: Analysis Cognitive Level: Knowledge NCLEX-RN Test Plan: Physiological Integrity; Reduction of Risk Potential ________________________________________ 3. An 84-year-old patient's skin is assessed to be

wrinkled, thin, and dry. These findings should be interpreted as related to: Your Answer: Increased vascularity of the skin. Correct Answer: Loss of elastin and a decrease in subcutaneous fat. Rationale: Several factors cause wrinkled, thin, dry skin. These include gradual loss of elastin; decreased subcutaneous fat; decreased number of epithelial cells providing a barrier, causing insensible loss of body fluids from the deeper layers of the skin; a lifetime of environmental trauma to the skin; increasing sedentary lifestyle; and decrease in mobility. Integrated Process: Nursing Process; Analysis Cognitive Level: Application NCLEX-RN Test Plan: Physiological Integrity; Reduction of Risk Potential ________________________________________ 4. A term used to describe discolored areas of aging skin due to damage to capillaries is: Your Answer: Actinic keratosis. Correct Answer: Senile purpura. Rationale: During the normal aging process, the vascularity of the dermis decreases and contributes to a paler complexion in the whiteskinned older person. The capillaries become thinner and more easily damaged, leading to bruised and discolored areas known as senile purpura. Integrated Process: Nursing Process; Assessment

Cognitive Level: Application NCLEX-RN Test Plan: Physiological Integrity; Reduction of Risk Potential ________________________________________ 5. Which of the following is accurate about skin cancer? Skin cancers that result primarily from sun exposure are: Your Answer: Hodgkin's disease. Correct Answer: Basal cell carcinomas. Rationale: Skin cancers that result primarily from sun exposure are basal cell carcinoma (BCC), squamous cell carcinoma (SCC), and malignant melanoma (MM). The risk for squamous cell carcinoma is strongly associated with chronic sun exposure but not with intermittent exposure. Hodgkin's disease is a cancer of the lymphatic system, and bronchogenic carcinoma is a lung cancer. Actinic keratotic lesions are precancerous skin lesions. Integrated Process: Nursing Process; Analysis Cognitive Level: Knowledge NCLEX-RN Test Plan: Physiological Integrity; Reduction of Risk Potential ________________________________________ 6. An elderly client is being assessed because of a rough, scaly, erythematous papule on the patient's body. Important teaching for this patient would include that this is most likely a (an): Your Answer: Erythematous actinic keratotic lesions.

Rationale: Erythematous actinic keratotic lesions are the most common type of precancerous lesion. They appear as painful rough, scaly, erythematous papules or plaque. The acrochordon, seborrheic keratoses, and senile lentigines do not become malignant. Integrated Process: Teaching/Learning Cognitive Level: Analysis NCLEX-RN Test Plan: Physiological Integrity; Reduction of Potential Risk ________________________________________ 7. Which of the following factors affect skin tolerance and enhance the occurrence of pressure ulcers? (Select all that apply.) Your Answer: Decreased serum albumin Correct Answers: Shearing Increased body weight Dramatic loss of collagen Rationale: Shearing, or the sliding of parallel surfaces against each other, occurs most commonly when the client slides down in the bed. Increased body weight and dramatic loss of collagen also could increase the risk of pressure ulcers. An increased serum albumin would not create a skin problem, but a decreased serum albumin would indicate a low level of protein stores and increased risk for impaired skin integrity. Integrated process: Nursing Process; Analysis

Cognitive level: Application NCLEX-RN Test Plan: Physiological Integrity: Basic Care and Comfort ________________________________________ 8. In order to prevent shearing injury to a 50-year-old terminally ill patient who is high-risk for skin damage, which of the following measures would be effective? (Select all that apply.) Your Answer: Keeping the patient's skin clean and dry Correct Answers: Encouraging the client to use a trapeze bar Keeping the patient's skin clean and dry Rationale: The high-risk patient for skin damage will need frequent position changes and clean, dry skin. A trapeze bar, turning sheet, and transfer board are specific interventions that can be used to prevent shearing injury to the skin. Integrated Process: Nursing Process; Planning Cognitive Level: Application NCLEX-RN Test Plan: Physiological Integrity; Basic Care and Comfort ________________________________________ 9. Which of the following statements is an accurate description for the inflammatory phase of wound development? The inflammatory phase: Your Answer: Begins soon after injury and continues for up to three weeks. Correct Answer: Is characterized by redness,

heat, pain, and edema or swelling. Rationale: The inflammatory phase of healing is characterized with the classic symptoms of inflammation, such as redness, heat, pain, and edema or swelling. The other answers relate to the different phases of wound development. Integrated Process: Nursing Process; Assessment Cognitive Level: Knowledge NCLEX-RN Test Plan: Physiological Integrity; Basic Care and Comfort ________________________________________ 10. A patient develops severe inflammation of the lower extremity. Which of the following foods should be included in the diet for this patient? Your Answer: Carrots and other yellow vegetables Correct Answer: Citrus fruits, protein, and fluids Rationale: The diet of the client with inflammation should be high in vitamin C, to aid in the synthesis of collagen; protein, to aid in the formation of blood cells and tissue; and fluids, to remove metabolic waste and rehydrate the client. Integrated Process: Nursing Process; Planning Cognitive Level: Synthesis NCLEX-RN Test Plan: Physiological Integrity; Basic Care and Comfort 1. How do the Scope and Standards of Gerontological Nursing Practice (ANA, 2004) guide professional

nursing practice? Your Answer: They are the legal standard for gerontological nursing practice. Rationale: The Scope and Standards deal with the professional obligations of the nurse when caring for the elderly; they are not intended as assessment measures. Boundaries of practice are addressed in the Nurse Practice Act of each state. Nursing Process: Implementation Cognitive Level: Analysis NCLEX-RN Test Plan: Safe, Effective Care Environment ________________________________________ 2. When an elderly client refuses to take the usual morning medications or get out of bed, the nurse must deal with ethical issues related to: Your Answer: Veracity. Correct Answer: Autonomy. Rationale: Autonomy relates to the clients right to self-determination and right to make decisions. Beneficence has to do with doing good, veracity deals with being truthful, and justice has to do with being fair. Nursing Process: Assessment Cognitive Level: Application NCLEX-RN Test Plan: Safe, Effective Care Environment ________________________________________ 3. What is the advanced practice registered nurse

(APRN) who is board certified (BC) in gerontology most likely able to do? Your Answer: Serve as an administrator of a nursing home. Correct Answer: Be employed as a nurse practitioner. Rationale: This role requires a graduate degree, and those with Masters degrees may seek board certification. Case managers are not required to be nurses, and nursing home administrators are required to have specialized education and certification in that discipline. Researchers are qualified to develop and test theories. Nursing Process: Implementation Cognitive Level: Analysis NCLEX-RN Test Plan: Safe, Effective Care Environment ________________________________________ 4. What information will the nurse use to respond to the elderly client who states, I will never go to a nursing home; I am going to die at home? Your Answer: Most people will need a brief stay in a nursing home at some time in their lives. Rationale: Current data indicate that over 40% of people will need at least a brief stay in a nursing home to recover from surgery, illness, or injury at some point in their lives. Other responses do not provide correct or supportive information. Nursing Process: Implementation Cognitive Process: Application

NCLEX-RN Test Plan: Psychosocial Integrity ________________________________________ 5. With what part of the functional health pattern assessment is the nurse most concerned when an elderly female is admitted to the hospital with deep jaundice? Your Answer: Nutritional-metabolic Rationale: Jaundice is caused by obstruction of one of the biliary ducts, and most frequently results in appetite changes. Health perceptionhealth maintenance and coping-stress management do not address functional changes. Elimination is not a priority concern when jaundice is present. Nursing Process: Assessment Cognitive Level: Application NCLEX-RN Test Plan: Health Promotion and Maintenance ________________________________________ 6. An appropriate goal for a elderly male who had a CVA with right-sided hemiplegia and a nursing diagnosis of impaired self care management would be: Your Answer: Requests that wife shave him daily in the morning. Correct Answer: Communicates need to use the urinal. Rationale: Requesting to use the urinal is indicative of self-care. Other options do not deal with the clients managing his physical care by

himself. Nursing Process: Planning Cognitive Level: Implementation NCLEX-RN Test Plan: Physiological Integrity ________________________________________ 7. The nurse expects to use the nursing process when first meeting an elderly client in the clinic. What is the priority nursing action before initiation of the nursing process? Your Answer: Ascertain the chief complaint. Correct Answer: Establish rapport with the client. Rationale: Effective use of the nursing process is dependent on the nurse first making the client feel comfortable and creating an environment of trust. Other actions would be appropriate after rapport is established. Nursing Process: Planning Cognitive Level: Analysis NCLEX-RN Test Plan: Safe, Effective Care Environment ________________________________________ 8. After providing basic care for an elderly client, the nurse concludes that hydration is a priority need. This decision represents which step of the nursing process? Your Answer: Assessment Correct Answer: Planning Rationale: Setting goals or outcomes is the planning phase of the nursing process, and should

involve the client. Assessment must occur prior to setting goals. Implementation is based on the outcomes; evaluation follows implementation and determines if outcomes have been achieved. Nursing Process: Planning Cognitive Level: Application NCLEX-RN Test Plan: Physiological Integrity ________________________________________ 9. Allowing an elderly female hospitalized for surgical repair of a fractured hip to take her medications with her morning cup of coffee is an example of: Your Answer: Providing culturally acceptable care. Correct Answer: An appropriate nursing intervention. Rationale: Clients can select the fluid of choice for taking medications unless there is a specific known contraindication for a particular medication. This intervention is not related to cultural practices or to evidence-based practice. Case management deals with planning for the hospital stay and post-hospital care needs. Nursing Process: Implementation Cognitive Level: Analysis NCLEX-RN Test Plan: Physiological Integrity ________________________________________ 10. Current demographic trends project a steadily increasing elderly population. What research focus recognizes this trend? Your Answer: Studies that demonstrate the

best timing to introduce the topic of durable powers of attorney for healthcare Correct Answer: Studies that examine the role strain of caregivers for the elderly Rationale: Increasing numbers of elderly mean there will be increasing numbers of caregivers who will experience stress. Studies need to be done to find ways to reduce the stress those individuals experience, whether they are family members or healthcare professionals. Other options deal with specific situations that are not related to population trends. Nursing Process: Evaluation Cognitive Level: Analysis NCLEX-RN Test Plan: Safe, Effective Care Environment

Das könnte Ihnen auch gefallen